Docsity
Docsity

Prepare for your exams
Prepare for your exams

Study with the several resources on Docsity


Earn points to download
Earn points to download

Earn points by helping other students or get them with a premium plan


Guidelines and tips
Guidelines and tips

TNCC Test Prep 8th, 9th Final Exam New 2024-2025 Latest Version with All Questions, Exams of Nursing

TNCC Test Prep 8th, 9th Final Exam New 2024-2025 Latest Version with All Questions and 100% Correct Answers

Typology: Exams

2023/2024

Available from 05/01/2024

exam-hut
exam-hut 🇺🇸

4.7

(3)

201 documents

1 / 113

Toggle sidebar

Related documents


Partial preview of the text

Download TNCC Test Prep 8th, 9th Final Exam New 2024-2025 Latest Version with All Questions and more Exams Nursing in PDF only on Docsity! TNCC Test Prep 8th, 9th Final Exam New 2024-2025 Latest Version with All Questions and 100% Correct Answers Which of the following is true about the log roll maneuver? A) it causes less spinal motion than the lift and slide manuever B) it is recommended for patients with unstable pelvic fractures C) it can worsen cord damage from an unstable spinal injury D) it does not increases the risk of life threatening hemorrhage from unstable injuries --- ------- Correct Answer --------- Answer: C A 20-year old male presents to the ED c/o severe lower abd pain after landing hard on the bicycle cross bars which performing an aerial BMX maneuver. Secondary assessment reveals lower abd tenderness and scrotal ecchymosis. Which of the following orders would the nurse questions? A) Fast exam B) CT scan C) straight cath for urine sample D) ice and elevation of the scrotum --------- Correct Answer --------- C) straight cath for urine sample Which of the following is possible complication of positive-pressure ventilation? A) worsening pneumothorax B) worsening flail chest C) reabsorption of pleural air D) negative intrapleural pressure --------- Correct Answer --------- A) worsening pneumothorax Which of the following is NOT considered goal-directed therapy of cardiogenic shock? A) controlled fluid boluses B) antidysrhythmic administration C) pericardiocentesis D) cardiac cath --------- Correct Answer --------- C) pericardiocentesis Tearing of the bridging veins is most frequently associated with which brain injury? A) epidural hematoma B) subdural hematoma C) diffuse axonal injury D) intracerebral hematoma ---------- Correct Answer --------- Answer: B EMS brings a patient who fell riding his bicycle. Using the American College of Surgeons screening guidelines, which assessment finding would prompt the nurse to prepare the patient for radiologic spine clearance? A) Alert with no neurologic deficits B) Multiple abrasions to the extremities C) Multiple requests of water D) Smell of alcohol on breath ---------- Correct Answer --------- Answer: D Which of the following occurs during the third impact of a motor vehicle crash? A) The driver of the vehicle collides with the steering wheel B) the vehicle collides with a tree C) the aorta is torn at its attachment with the ligamentum arteriosum D) the airbag deploys and strikes the front seat passenger ---------- Correct Answer ------ --- Answer: C If a patient has received multiple transfusions of banked blood preserved with citrate, which electrolyte is most likely to drop and require supplementation? A) potassium B) magnesium C) sodium D) calcium ---------- Correct Answer --------- Answer: D A trauma patient is en route to a rural ED. Radiology notifies the charge nurse that the CT scanner will be out of service for several of hours. The team gathers to plan accordingly. Which of the following terms best describes this trauma teams communication? A) Brief B) loop C) debrief D) huddle ---------- Correct Answer --------- Answer: D The nurse is caring for a 120 kg male is brought in after a warehouse fire and is calculating the patient's fluid resuscitation needs. He has painful red blistering to the entire surface of both upper extremities and superficial burns to the anterior chest. Using the modified Lund and Browder chart to calculate the total BSA burned, how much IV fluids would be administered in the first 8 hours? (upper arm 4+4 / lower arm 3+3 / hand 2.5+2.5) = 19% TBSA. 120 kg x (2 mL) x (19% TBSA) = 4560 mL 4560 / 2 (half of fluids given in first 8 hours) = 2280 mL performed by the surgical team to rapidly identify the presence of hemorrhage in patients who are hemodynamically unstable after trauma. Which of the following is an expected finding in a patient with a thoracostomy connected to a chest drainage system? A) output of 200 mL/hr B) tubing clamp closed for transport C) dependent loops in the tubing to promote drainage D) fluctuations in the water serial chamber ---------- Correct Answer --------- Answer: D Which of the following patients warrants referral to a burn center? A) a 21- year old female with a partial thickness burn to the right forearm B) a 40-year old hypertensive male with a superficial burn to the back C) a 52-year old diabetic male with partial thickness burn to the left lower leg D) a 35-year old hyperlipidemic female with superficial burns to the anterior thorax. ------ ---- Correct Answer --------- Answer: C A patient has been in the ED for several hours waiting to be admitted. He sustained multiple rib fractures and a femur fracture after a fall. He has been awake, alert, and c/o leg pain. His wife reported suddenly becoming anxious and confused. Upon reassessment, the patient is restless, with respiratory distress and petechiae to his neck. the patient is exhibiting signs of symptoms commonly associated with which of the following conditions? A) acute lung injury B) fat embolism C) PTX D) pulmonary contusion ---------- Correct Answer --------- Answer: B An adult patient who sustained a severe head trauma has been intubated and is being manually ventilated via a bag-mask device at a rate of 18 breaths/minute. The patient has received one intravenous fluid bolus of 500 mL of warmed isotonic crystalloid solution. The PaCO2 is 30 mm Hg (4.0 kPa), and the pulse oximetry is 92%. BP is 142/70 mm Hg. What is the most important intervention to manage the cerebral blood flow? A. Decrease the rate of manual ventilation B. Initiate another fluid bolus. C. Recheck endotracheal tube placement. D. Increase the amount of oxygen delivered. --------- Correct Answer ---------- A. Decrease the rate of manual ventilation Which of the following situations could cause functional grief? A. Inability to live at home B. Amputation of a limb C. Loss of one's self-image D. Destruction of the patient's car --------- Correct Answer ---------- B. Amputation of a limb Rationale: Preparation and Ongoing Care\Psychosocial Aspects Functional grief relates to the loss of body function or body parts such as amputation of a limb, paralysis, or loss of sight. What is the most important consideration during the initial assessment when caring for an older adult who has sustained serious injuries? A. They are likely to be fearful in the emergency department B. Medical history including current medications C. Availability of support systems after discharge D. Accessibility to a primary care physician --------- Correct Answer ---------- B. Medical history including current medications 4. You are caring for a patient who was thrown from a bike and was not wearing a helmet. While performing the head-to-toe assessment, you note clear drainage from the right ear. Which of the following is the most appropriate next step? A. Clean the ear with a cotton-tipped applicator. B. Pack the ear with gauze. C. Notify the physician D. Document and continue the exam. --------- Correct Answer ---------- C. Notify the physician 5. When should the definitive calculation for intravenous fluid resuscitation rate be performed for a patient with burns? A. As soon as the patient arrives B. After removal of clothing C. Only at a burn center D. During the circulation assessment --------- Correct Answer ---------- B. After removal of clothing Rationale: Musculoskeletal and Wounds\Burns Some fluid will be given initially, but an accurate fluid total is based on percentage of total body surface area which requires a good skin assessment and is calculated after clothing is removed. 6. In a motor vehicle collision, which injury pathway is most likely to increase the patient's morbidity and mortality? A. Rotational B. Ejection C. Lateral D. Rollover --------- Correct Answer ---------- B. Ejection 7. A patient with a lower extremity fracture complains of severe pain and tightness in his calf, minimally relieved by pain medications. Which of the following is the priority nursing intervention? A. Elevating the leg above the level of the heart B. Repositioning the leg and applying ice C. Elevating the leg to the level of the heart D. Preparing the patient for ultrasound of the leg --------- Correct Answer ---------- C. Elevating the leg to the level of the heart Rationale: Musculoskeletal and Wounds\Musculoskeletal This patient is exhibiting signs of possible compartment syndrome which is a dangerous complication of fractured extremities. The pain is often out of proportion to the injury and might not respond to pain medications. Elevate the limb to the level of the heart to decrease dependent edema but not above the heart, which can cause increased venous congestion and pressure within the compartment. 8. A teenaged patient presents to the emergency department with left arm pain after a ground level fall. The patient identifies as transgender and shares they are homeless. The ED staff are concerned the patient is experiencing human trafficking. What is most consistent with human trafficking? A. The individual appears well nourished. B. Those who are being trafficked rarely seek medical care. C. It is infrequently associated with substance abuse. D. Those experiencing human trafficking rarely self-disclose. --------- Correct Answer ---- ------ D. Those experiencing human trafficking rarely self-disclose. 9. An intubated trauma patient has just been transported back from CT scan. Upon arrival to their room in the emergency department, resistance is noted with bag-mask ventilations and auscultation reveals unequal breath sounds. What is the most appropriate initial intervention for this patient? A. Place the patient back on the ventilator. B. Extubated the patient. C. Reposition the endotracheal tube. D. Suction the endotracheal tube. --------- Correct Answer ---------- C. Reposition the endotracheal tube. 10. Which of the following assessment findings differentiates a tension pneumothorax from a simple pneumothorax? A. Increased work of breathing B. Unilaterally diminished breath sounds C. Tachycardia D. Hypotension --------- Correct Answer ---------- D. Hypotension 11. A patient is brought to the emergency department of a rural hospital following a high-speed motor vehicle collision. When significant abdominal and pelvic injuries are noted in the primary survey, what is the priority intervention? A. Oculomotor nerve palsy B. Globe rupture C. Retrobulbar hematoma D. Retinal detachment --------- Correct Answer ---------- B. Globe rupture Rationale: Head and Torso Trauma\Head Trauma The globe of the eye consists of multiple layers. The sclera is the white outer layer, while the cornea covers the iris and pupil. When a full thickness injury occurs to the cornea, sclera, or both, the globe will rupture, causing the shape of the pupil to become irregular or teardrop-shaped 21. An adult arrives at the emergency department with superficial burns to the extremities following a house fire. The patient is reporting a headache with nausea and is drowsy and confused. What is the most likely cause of these symptoms? A. Capillary leak syndrome B. Rhabdomyolysis C. Carbon monoxide poisoning D. Hypothermia --------- Correct Answer ---------- C. Carbon monoxide poisoning 22. An adult patient is being treated in the emergency department after falling 30 feet and landing on their head. The patient has been intubated and initial stabilization efforts are ongoing. The vital signs are BP 70/40 mm Hg, HR 44 beats/minute, and respirations are being assisted at 16 breath/minute. The patient's skin is warm, dry, and of a normal color. These findings are most consistent with what type of shock? A. Spinal B. Hypovolemic C. Obstructive D. Neurogenic --------- Correct Answer ---------- D. Neurogenic 23. A patient with a chest tube is being transported to the intensive care unit and fluctuation is noted in the water seal chamber during inspiration and expiration. What is the best action for the nurse to take? A. Clamp the chest tube B. Return to the emergency department C. Assist ventilations with bag-mask device D. Continue to the intensive care unit --------- Correct Answer ---------- D. Continue to the intensive care unit 24. 1. Which one of the following characteristics is found in high performing teams? A. Interdisciplinary collaboration B. Provide on-the-job training C. Work individually D. Minimal feedback to decrease confusion --------- Correct Answer ---------- A. Interdisciplinary collaboration 25. 1. A severely injured patient has been intubated and is being mechanically ventilated. The patient has received a balanced resuscitation including multiple blood products. Under which circumstance will it be harder for the hemoglobin to release oxygen to the tissues? A. Decreased pH B. Elevated carbon dioxide level C. Decreased body temperature D. Increased metabolic demand Rationale: Primary Assessment\Airway and Ventilation If the blood is not sufficiently warmed when transfused, it can produce hypothermia. In the oxyhemoglobin dissociation curve, a shift to the left increases the affinity of hemoglobin for oxygen making it harder to release for use by the tissues. Factors that can cause this shift include an elevated pH, decreased carbon dioxide, decreased temperature, and low metabolic demand. --------- Correct Answer ---------- C. Decreased body temperature Why is a measure of serum lactate obtained in the initial assessment of the trauma patient? a) to measure oxygenation and ventilation b) to quantify the base deficit for the adequacy of cellular perfusion c) to gauge end-organ perfusion and tissue hypoxia d) to determine the underlying cause of shock ---------- Correct Answer --------- Answer: c A trauma patient is restless and repeatedly asking "where am I?" vital signs upon arrival were BP 100/60 mm Hg, HR 96 beats/min, and RR 24 breaths/min. Her skin is cool and dry. Current vital signs are BP 104/84mm Hg, HR 108, RR 28 breaths/min. The patient is demonstrating signs and symptoms of which stage of shock? A) compensated B) Progressive C) irreversible D) decompensated ---------- Correct Answer --------- Answer: A An elderly patient with a history of anticoagulant use presents after a fall at home that day. she denies any loss of consciousness. She has a hematoma to her forehead and complains of headache, dizziness, and nausea. Which is a most likely cause of her symptoms? A) epidural hematoma B) diffuse axonal injury C) post-concussive syndrome D) subdural hematoma ---------- Correct Answer --------- Answer: D A passenger is brought to the emergency department of a rural hospital following a high-speed MVC. When significant abdominal and pelvic injuries are noted in the primary survey, which of the following is the priority interventions? A) initiate transfer to a trauma center B) provide report to the operating room nurse C) Obtained imaging studies D) Place a gastric tube ---------- Correct Answer --------- Answer: A Which of the following injuries is LEAST likely to be promptly identified? A) spleen B) lung C) bowel D) brain ---------- Correct Answer --------- Answer: C Patients with a crush injury should be monitored for which of the following conditions? A) Hypernatremia B) Hypercalcemia C) Dysrhythmias D) polyuria ---------- Correct Answer --------- Answer: C What finding raises suspicion of complete spinal cord injury? A) Weakness in the lower extremities B) Priapism C) voluntary anal sphincter tone D) intact reflexes distal to the injury ---------- Correct Answer --------- Answer: B A patient with a complete spinal cord injury in neurogenic shock will demonstrate hypotension and which other clinical signs? A) Bradycardia and ipsilateral absences of motor function B) Tachycardia and respiratory depression C) Tachycardia and absent motor function below the level of injury D) Bradycardia and absent motor function below the level of the injury ---------- Correct Answer --------- Answer: D A 5-year old child presents to the ED with bruises to the upper arms and buttocks in various stages of healing and multiple small, clean, round burns to the back. There are no abnormalities found based on the pediatric assessment triangle or primary survey. Which of the following is the priority nursing intervention? A) report your suspicion of the maltreatment in accordance with local regulations B) apply ice to the bruises and consult wound care C) engage in therapeutic communication to determine the MOI D) provide the family with injury prevention resourse ---------- Correct Answer --------- Answer: A A patient with lower extremity fracture complains of severe pain and tightness in his calf, minimally relieved by pain medications. Which of the following is the priority nursing intervention? A) elevating the extremity above the level of the heart A) oculomotor nerve palsy B) globe rupture C) uncal herniation D) retinal detachment ---------- Correct Answer --------- Answer: B EMS arrives with the intoxicated driver of a car involved in a MVC. EMS reports significant damage to the drivers side of the car. The patient is asking to have the cervical color removed. when is it appropriate to remove the cervical collar? A) after a physical examination if the patient has no radiologic abnormalities on a computed tomography scan B) after physician evaluation if the patient has not neurologic abnormalities on exam C) after palpation of the spine if the patient has no point tenderness the the vertebral column D) after physical examination if the patient has not neck pain with movement ---------- Correct Answer --------- Answer: A Which is the effect of hypothermia on the oxyhemoglobin dissociation curve? A) Hemoglobin does not readily release oxygen for use by the tissues B) The amount of oxygen available to the tissues increases C) Tissue oxygenation (PaO2) increases D) Hemoglobin molecule saturation (SaO2) decreases ---------- Correct Answer --------- Answer: A A 20-year old male presents to the ED c/o severe lower abd pain after landing hard on the bicycle cross bars which performing an aerial BMX maneuver. Secondary assessment reveals lower abd tenderness and scrotal ecchymosis. Which of the following orders would the nurse questions? A) Fast exam B) CT scan C) straight cath for urine sample D) ice and elevation of the scrotum ---------- Correct Answer --------- Answer: C You are treating a 27-year old male in respiratory distress who was involved in a house fire. Calculating TBSA burned is deferred d/t the need for emergent intubation. At what rate should you begin fluid resuscitation? A) 1000 mL/hr B) 500 mL/hr C) 250 mL/hr D) 125 mL/hr ---------- Correct Answer --------- Answer: B Which of the following is possible complication of positive-pressure ventilation? A) worsening pneumothorax B) worsening flail chest C) reabsorption of pleural air D) negative intrapleural pressure ---------- Correct Answer --------- Answer: A Which of the following is NOT considered goal-directed therapy of cardiogenic shock? A) controlled fluid boluses B) antidysrhythmic administration C) pericardiocentesis D) cardiac cath ---------- Correct Answer --------- Answer: C What bedside monitoring parameters are used to assess for adequacy of oxygenation and effectiveness of ventilation? A) pulse oximetry and capnogaphy B) respiratory rate and capnography C) pulse oximetry and respiratory rate D) capnography and capnometry ---------- Correct Answer --------- Answer: A Which if the following values indicates the need for alcohol withdrawal interventions? A) CIWA-Ar of 36 B) GCS 13 C) ETCo2 of 48 mm Hg D) heart rate of 45 beats/min ---------- Correct Answer --------- Answer: A An unresponsive trauma patient has an oropharyngeal airway in place, shallow and labored respiratory, and dusky skin. the trauma team has administered medications for drug assisted intubation and attempted intubation but was unsuccessful. What is the most appropriate immediate next step? A) Ventilate with a BVM B) Prepare for cricothyroidotomy C) administer reversal medications D) contact anesthesia for assistance ---------- Correct Answer --------- Answer: A The trauma nurse knows that placing a bariatric patients in a "ramped position" provides better visualization during the insertion of which device? A) Intraosseous line B) orogastric tube C) ET tube D) urinary catheter ---------- Correct Answer --------- Answer: C You are caring for a patient who was involved in a MVA and is 32 weeks pregnant. Findings of your secondary survey include abd pain on palpation, fundal high at the costal margin, and some dark bloody show. Varying, accelerations and decelerations are noted on the cardiotocography. These findings are most consistent with which of the following? A) placental abruption B) preterm labor C) uterine rupture D) fetal demise ---------- Correct Answer --------- Answer: A Expedite transfer to the closest trauma center ---------- Correct Answer --------- A 56 y/o M pt involved in a motor vehicle crash is brought to the ED of a rural critical access facility. He complains of neck pain, SOB, and diffuse abd pain. His GCS is 15. His VS: BP 98/71, HR 125, RR 26, SpO2 94% on high-flow O2 via NRB mask. Which of the following is the priority intervention for this patient? a pertinent medical hx is crucial ---------- Correct Answer --------- Which of the following considerations is the most important when caring for a geriatric trauma pt? Mitigation ---------- Correct Answer --------- Following a review of recent drills and a real disaster event, a hospital has identified deficiencies and is taking steps to minimize the impact of a future disaster . Which phase of the disaster life cycle does this describe? Alcohol All tools and guidelines require the patient to be awake, alert, not under the influence of any substances, and without neurologic abnormalities ---------- Correct Answer --------- EMS brings a pt who fell while riding his bicycle. Using the American College of Surgeons screening guidelines, which assessment finding would prompt the RN to prepare the pt for a radiologic spine clearance? hemoglobin does not readily release O2 for use by the tissues ---------- Correct Answer - -------- What is the effect of hypothermia on the oxyhemoglobin dissociation curve? acidosis ---------- Correct Answer --------- Which of the following is a component of the trauma triad of death? Complete ---------- Correct Answer --------- EMS brings a pt from MVC. VS: BP 90/49, HR 48, RR 12, temp 97.2F (36.2 C). The pt exhibits urinary incontinence and priapism. These assessment findings are most consistent with which of the following types of spinal cord injury? flucuation in the water seal chamber ---------- Correct Answer --------- Which of the following is an expected finding in a pt with a tube thoracstomy connected to a chest drainage system? and buttocks in various stages of healing and multiple small, clean, round burns to the back. There are no abnormalities found based on the pediatric assessment triangle or primary survey. Which of the following is the priority survey. Which of the following is the priority nursing intervention? to guage end-organ perfusion and tissue hypoxia ---------- Correct Answer --------- Why is a measure of serum lactate obtained in the initial assessment of a trauma patient? elevating the extremity to the level of the heart ---------- Correct Answer --------- A pt with a lower extremity fracture complains of severe pain and tightness in his calf, minimally by pain medications. Which of the following is the priority nursing intervention? velocity ---------- Correct Answer --------- What factor contributes most to the kinetic energy of a body in motion? subdural hematoma ---------- Correct Answer --------- An elderly patient with a history of anticoagulant use presents after a fall at home today. She denies any loss of consciousness. She has a hematoma to her forehead and complains of headache, dizziness, and nausea. What is the most likely cause of her symptoms? fat embolism ---------- Correct Answer --------- a pt has been in the ED for several hrs waiting to be admitted. He sustained multiple rib fractures and a femur fracture after a fall. He has been awake, alert, and complaining of leg pain. His wife reported that he suddenly became anxious and confused. Upon reassessment, the pt is restless with respiratory distress and petechiae to his neck. The pt is exhibiting s/sx most commonly associated with which of the following conditions? Late signs of increased intracranial pressure include: dilated nonreactive pupils, unresponsive to verbal or painful stimuli, abnormal motor posturing, and Cushing response (widening pulse pressure, reflex bradycardia, decreased respiratory effort) ---- ------ Correct Answer --------- Which of the following is a late sign of increased intracranial pressure? serial FAST exams ---------- Correct Answer --------- a 49 y/o restrained driver involved in a MVC presents to the trauma center complaining of abd, pelvic, and bilateral lower extremity pain. VS are stable. The nurse can anticipate all of these after a negative FAST exam EXCEPT which of the following? pericardiocentesis ---------- Correct Answer --------- Which of the following is NOT considered goal-directed therapy for cardiogenic shock? endotracheal tube ---------- Correct Answer --------- The trauma nurse knows that placing a bariatric patient in a ramped position providers better visualization during the insertion of which device? increased work of breathing Assessment findings associated with tension pneumothorax include anxiety, severe restlessness, severe respiratory distress, and absent breath sounds on the injured side ---------- Correct Answer --------- which of the following assessment findings differentiates a tension pneumothorax from a simple pneumothorax? calcium ---------- Correct Answer --------- if a pt has received multiple transfusions of banked blood preserved with citrate, which electrolyte is most likely to drop and require supplementation? identifying individuals who made mistakes during the traumatic event ---------- Correct Answer --------- Which of the following is NOT considered a benefit of debriefings? 500 mL/hr ---------- Correct Answer --------- You are treating a 27 y/o M in respiratory distress who was involved in a house fire. Calculating TBSA burned is deferred due to the need for emergent intubation. At what rate should you begin fluid resuscitation? advanced age ---------- Correct Answer --------- Which of the following is most likely to contribute to inadequate oxygenation and ventilation? a 52 y/o diabetic male with a partial thickness burn to the left lower leg ---------- Correct Answer --------- Which of the following patients warrants referral to a burn center? dressing removal ---------- Correct Answer --------- A patient arrives with a large open chest wound after being assaulted with a machete. Prehospital providers placed a nonporous dressing over the chest wound and taped it on three sides. he is now showing signs of anxiety, restlessness, severe respiratory distress, cyanosis and decreasing blood pressure. Which of the following is the MOST appropriate immediate intervention? after a physical examination if the pt has no radiologic abnormalities on CT ---------- Correct Answer --------- EMS arrives with the intoxicated driver of a car involved in a MVC. EMS reports significant damage to the drivers side of the car. The pt is asking to have the cervical collar removed. When it is appropriate to remove the cervical collar? MARCH ---------- Correct Answer --------- Which of the following mnemonics can help the nurse prioritize care for a trauma patient with massive uncontrolled hemorrhage? What is kinematics? ---------- Correct Answer --------- A branch of mechanics (energy transfer) that refers to motion and does not consider the concepts of force and mass of the object or body. What is Newton's First Law? ---------- Correct Answer --------- A body at rest will remain at rest. A body in motion will remain in motion until acted on by an outside force. What is the Law of Conservation of Energy? ---------- Correct Answer --------- Energy can neither be created nor destroyed. It is only changed from one form to another. What is Newton's Second Law? ---------- Correct Answer --------- Force equals mass multiplied by acceleration of deceleration. What is kinetic energy (KE)? ---------- Correct Answer --------- KE equals 1/2 the mass (M) multiplied by the velocity squared. What is the Mnemonic for the Initial Assessment? ---------- Correct Answer --------- A = Airway with simultaneous cervical spine protection B = Breathing C = Circulation D = Disability (neurologic status) E = Expose/Environmental controls (remove clothing and keep the patient warm) What is the Mnemonic for the Secondary Assessment? ---------- Correct Answer --------- F = Full set of VS/Focused adjuncts (includes cardiac monitor, urinary catheter, and gastric tube)/Family presence G = Give comfort measures (verbal reassurance, touch, and pharmacologic and nonpharmacologic management of pain). H = Hx and Head-to-toe assessment I = Inspect posterior surfaces Where do you listen to auscultate breath sounds? ---------- Correct Answer --------- Auscultate the lungs bilaterally at the second intercostal space midclavicular line and at the fifth intercostal space at the anterior axillary line. What are the late signs of breathing compromise? ---------- Correct Answer --------- - Tracheal deviation - JVD What are signs of ineffective breathing? ---------- Correct Answer --------- - AMS - Cyanosis, especially around the mouth - Asymmetric expansion of chest wall - Paradoxical movement of the chest wall during inspiration and expiration - Use of accessory muscles or abdominal muscles or both or diaphragmatic breathing - Sucking chest wounds - Absent or diminished breath sounds - Administer O2 via NRB or assist ventilations with a bag-mask device, as indicated - Anticipate definitive airway management to support ventilation. Upon initial assessment, what type of oxygen should be used for a pt breathing effectively? ---------- Correct Answer --------- A tight-fitting nonrebreather mask at 12-15 lpm. - LOC - Neurologic injury - Spinal Cord Injury - Intracranial Injury - Blunt trauma - Pain caused by rib fractures - Penetrating Trauma - Preexisting hx of respiratory diseases - Increased age What medications are used during intubation? ---------- Correct Answer --------- LOAD Mnemonic: L = Lidocaine O = Opioids A = Atropine D = Defasiculating agents What are the Rapid Sequence Intubation Steps? ---------- Correct Answer --------- PREPARATION: - gather equipment, staffing, etc. PREOXYGENATION: - Use 100% O2 (prevent risk of aspiration). PRETREATMENT: - Decrease S/E's of intubation PARALYSIS WITH INDUCTION: - Pt has LOC, then administer neuromuscular blocking agent PROTECTION AND POSITIONING: - Apply pressure over cricoid cartilage (minimizes likelihood of vomiting and aspiration PLACEMENT WITH PROOF - Each attempt NOT to exceed 30 seconds, max of 3 attempts. Ventilate pt 30-60 seconds between attempts. - After intubation, inflate the cuff - Confirm tube placement w/exhaled CO2 detector. POSTINTUBATION MANAGEMENT: - Secure ET tube - Set ventilator settings - Obtain Chest x-ray - Continue to medicate - Recheck VS and pulse oxtimetry What is a Combitube? ---------- Correct Answer --------- A dual-lumen, dual-cuff airway that can be placed blindly into the esophagus to establish an airway. If inadvertently placed into trachea, it can be used as a temporary ET tube. There are only two sizes: small adult and larger adult. What is a Laryngeal Mask Airway? ---------- Correct Answer --------- Looks like an ET tube but is equipped with an inflatable, elliptical, silicone rubber collar at the distal end. It is designed to cover the supraglottic area. ILMA, does not require laryngoscopy and visualization of the chords. What is Needle Cricothyrotomy ---------- Correct Answer --------- Percutaneous transtracheal ventilation. (temporary) Complications include: - inadequate ventilation causing hypoxia - hematoma formation - esophageal perforation - aspiration - thyroid perforation - subcutaneous emphysema What is Surgical Cricothyrotomy? ---------- Correct Answer --------- Making an incision in cricothyroid membrane and placing a cuffed endo or trach tube into trachea. This is indicated when other methods of airway management have failed and pt cannot be adequately ventilated and oxygenated. Complications include: - Aspiration - Hemorrhage or hematoma formation or both - Lac to trachea or esophagus - Creation of a false passage - Laryngeal stenosis How do you confirm ET Tube/Alternative Airway Placement? ---------- Correct Answer --- ------ - Visualization of the chords - Using bronchoscope to confirm placement - Listening to breath sounds over the epigastrum and chest walls while ventilating the pt - CO2 detector - Esophageal detection device - Chest x-ray How do you inspect the chest for adequate ventilation? ---------- Correct Answer --------- Observe: - mental status - RR and pattern - chest wall symmetry - any injuries - patient's skin color (cyanosis?) - JVD or tracheal deviation? (Tension pneumothorax) What are you looking for when auscultating lung sounds? ---------- Correct Answer ------- -- Absence of BS: - Pneumothorax - Hemothorax - Airway Obstruction Diminished BS: - Splinting or shallow BS may be a result of pain What are you looking for when percussing the chest? ---------- Correct Answer --------- Dullness: - hemothorax Hyperresonance - Pneumothorax What are you looking for when palpating the chest wall, clavicles and neck? ---------- Correct Answer --------- - Tenderness - Swelling - subcutaneous emphysema - step-off deformities = These may indicate: esophageal, pleural, tracheal or bronchial injuries. Palpate trachea above suprasternal notch. Tracheal deviation may indicate a tension pneumothorax or massive hemothorax. What is the DOPE mnemonic? ---------- Correct Answer --------- D - Displaced tube O - Obstruction: Check secretions or pt biting tube P - Pneumothorax: Condition may occur from original trauma or barotrauma from ventilator E - Equipment failure: pt may have become detached from equipment or there's a kink in the tubing Explain Hypovolemic Shock. ---------- Correct Answer --------- Most common to affect a trauma pt cause by hypovolemia.. Hypovolemia, a decrease in amount of circulating blood volume, may result from significant loss of whole blood because of hemorrhage or from loss of semipermeable integrity of cellular membrane leading to leakage of plasma and protein from intravascular space to the interstitial space (as in a burn). Some causes: - Blood loss - Burns, etc. Explain Cardiogenic Shock. ---------- Correct Answer --------- Syndrome that results from ineffective perfusion caused by ineffective perfusion caused by inadequate contractility of cardiac muscle. Some causes: - MI - Membranes of lysosomes breakdown within cells and release digestive enzymes that cause intracellular damage. How would you assess someone in hypovolemic shock? ---------- Correct Answer --------- (Use Initial Assessment) and then: Inspect: - LOC - Rate and quality of respirations - External bleeding? - Skin color and moisture - Assess jugular veins and peripheral veins Auscultate: - BP - Pulse pressure - Breath sounds - Heart sounds - Bowel sounds Percuss: - Chest and abdomen Palpate: - Central pulse (carotid or femoral) - Positive inotropic effect (force of contraction) may be evidence by a bounding central pulse - Palpate peripheral pulses - Palpate skin temp and moisture Diagnostic Procedures: - Xrays and other studies - Labs Planning and Implementation - Oxygen - IV's with warmed replacement fluids - Control external bleeding with direct pressure - Elevate LE's - NGT - Foley - Monitor and pulse oximeter - Monitor for development of coagulopathies - Surgery? ICP is a reflection of what three volumes? What happens when one increases? ---------- Correct Answer --------- 1. Brain 2. CSF 3. Blood within the nonexpansible cranial vault As volume of one increases, the volume of another decreases to maintain ICP within normal range. As ICP rises, CPP decreases, leading to cerebral ischemia and potential for hypoxia and lethal secondary insult. Hypotensive pt w/marginally elevated ICP can be harmful. Slightly elevated BP could protect against brain ischemia in a pt with high ICP. Cerebral ischemia can lead to increased concentration of CO2 and decreased concentration of O2 in cerebral vessels. CO2 dilates cerebral blood vessels = increase blood volume and ICP. What are the early signs and symptoms of increased ICP? ---------- Correct Answer ------ --- - Headache - N/V - Amnesia regarding events around the injury - Altered LOC - Restlessness, drowsiness, changes in speech, or loss of judgement What are the late observable signs of symptoms of increased ICP? ---------- Correct Answer --------- - Dilated, nonreactive pupil - Unresponsiveness to verbal or painful stimuli - Abnormal motor posturing patterns - Widening pulse pressure - Increased systolic blood pressure - Changes in RR and pattern - Bradycardia What is Cushing's phenomenon or Cushing's Reflex? ---------- Correct Answer --------- Triad of progressive HTN, bradycardia and diminished respiratory effort. What are the two types of herniation that occurs with ICP? ---------- Correct Answer ------ --- 1. Uncal herniation 2. Central or transtentorial herniation Why does herniation occur? What are the symptoms? ---------- Correct Answer --------- Because of uncontrolled increases in ICP. S/E's - Unilateral or bilateral pupillary dilation - AsyDimmetric pupillary reactivity - Abnormal motor posturing - Other evidence of neurologic deterioration Define uncal herniation. ---------- Correct Answer --------- The uncus (medial aspect of the temporal lobe) is displaced over the tentorium into the posterior fossa. This herniation is the more common of the two types of herniation syndromes. Define central or transtentorial herniation. ---------- Correct Answer --------- A downward movement of the cerebral hemispheres with herniation of the diencephalon and midbrain through the elongated gap of the tentorium. Disruptions of the bony structures of the skull can result in what? ---------- Correct Answer --------- Displaced or nondisplaced fx's causing CSF leakage b/c of lac to the dura mater, creating a passage for CSF. CSF leaks through the nose (rhinorrhea) or the ears (otorrhea). A potential entrance for invading bacteria. Also: meningitis or encephalitis or brain abscess Define Minor Head Trauma. ---------- Correct Answer --------- GCS 13-15 Define Moderate Head Trauma ---------- Correct Answer --------- Postresuscitative state with GCS 9-13. Define Severe Head Trauma. ---------- Correct Answer --------- Postresuscitative state with GCS score of 8 or less. What is a concussion and its signs and symptoms? ---------- Correct Answer --------- A temporary change in neurologic function that may occur as a result of minor head trauma. S/S: - Transient LOC - H/A - Confusion and disorientation - Dizziness - N/V - Loss of memory - Difficulty with concentration - Irritability - Fatigue What are the signs and symptoms of postconcussive syndrome? ---------- Correct Answer --------- - Persistent H/A - Dizziness - Nausea - Memory impairment - Attention deficit - Irritability - Insomnia - Impaired judgement - Loss of libido - Anxiety - Depression INSPECTION: - Assess airway - RR, pattern and effort - Assess pupil size and response to light - Unilateral fixed and dilated pupil = oculomotor nerve compression from increased ICP + herniation syndrome - Bilateral fixed and pinpoint pupils indicate a pontine lesion or effects of opiates - Mildly dilated pupil w/sluggish response may be early sign of herniation syndrome - Widely dilated pupil occasionally occurs w/direct trauma to globe of eye - Determine if pt uses eye meds - Abnormal posturing? - Inspect craniofacial area for ecchymosis/contusions - Periorbital ecchymosis - Mastoid's process ecchymosis - Blood behind tympanic membrane - Inspect nose and ears for drainage - Drng present w/out blood, test drng w/chemical reagant strip. Presence of glucose indicated drng of CSF - If drng present and mixed with blood, test by placing drop of fluid on linen or gauze. If a light oute What are signs of a serious eye injury? ---------- Correct Answer --------- - Visual disturbances - Pain - Redness and ecchymosis of the eye - Periorbital ecchymosis - Increased intraocular pressure What is hyphema and its S/S? ---------- Correct Answer --------- Accumulation of blood, mainly RBC's that disperse and layer within the anterior chamber. A severe hymphema obscures entire anterior chamber + will diminish visual acuity severely or completely. Injuries are graded on amount of blood in chamber (Grades I-IV). S/S: - Blood in anterior chamber - Deep, aching pain - Mild to severe diminished visual acuity - Increased intraocular pressure What are s/s of chemical burns to the eye? ---------- Correct Answer --------- Chemical injuries require immediate intervention if it is to be preserved. S/S: - Pain - Corneal Opacification - Coexisting chemical burn and swelling of lids What are S/S of penetrating trauma/open or ruptured globe? ---------- Correct Answer --- ------ - Marked visual impairments - Extrusion of intraocular contents - Flattened or shallow anterior chamber - Subconjunctival hemorrhage, hyphema - Decreased intraocular pressure - Restriction of extraocular movements What are the S/S of orbital fracture (orbital blowout fracture)? ---------- Correct Answer -- ------- - Diplopia (double vision) - Loss of vision - Altered extraocular eye movements - Enophthalmos (displacement of the eye backward into the socket) - Subconjunctival hemorrhage or ecchymosis of the eyelid - Infraorbital pain or loss of sensation - Orbital bony deformity What is LeFort I fracture and its S/S? ---------- Correct Answer --------- Transverse maxillary fx that occurs above level of teeth and results in separation of teeth from rest of maxilla. S/S: - Slight swelling of maxillary area - Possible lip lac's or fractured teeth - Independent movement of the maxilla from rest of face - Malocclusion What is LeFort II fracture and its S/S? ---------- Correct Answer --------- Pyramidal maxillary fx=middle facial area. Apex of fx transverses bridge of nose. Two lateral fx's of pyramid extend through the lacrimal bone of the face and ethmoid bone of skull into the median portion of both orbits. Base of the fx extends above level of the upper teeth into maxilla. CSF leak is possible. S/S: - Massive facial edema - Nasal swelling w/obvious fx of nasal bones - Malocclusion - CSF rhinorrhea What is LeFort III fracture and its S/S? ---------- Correct Answer --------- Complete craniofacial separation involving maxilla, zygoma and bones of cranial base. This fx is frequently associated w/leakage of CSF and fx mandible. S/S: - Massive facial edema - Mobility and depression of zygomatic bones - Ecchymosis - Anesthesia of the cheek - Diplopia - Open bite or malocclusion - CSF rhinorrhea What are the mandibular fracture S/S? ---------- Correct Answer --------- - Malocclusion - Inability to open the mouth (trismus) - Pain, especially on movement - Facial asymmetry and a palpable step-off deformity - Edema or hematoma formation at the fracture site - Blood behind, ruptured, tympanic membrane - Anesthesia of the lower lip What are neck injury S/S? ---------- Correct Answer --------- - Dyspnea - Hemoptysis (coughing up blood) - Subcutaneous emphysema in neck, face, or suprasternal area - Decreased or absent breath sounds - Penetrating wounds or impaled objects - Pulsatile or expanding hematoma - Loss of normal anatomic prominence of the laryngeal region - Bruits - Active external bleeding - Neurologic deficit, such as aphasia or hemiplegia - Cranial nerve deficits - Facial sensory or motor nerve deficits - Dysphonia (hoarseness) - Dysphagia (difficulty swallowing) How would you assess a patient with ocular, maxillofacial and neck trauma? ---------- Correct Answer --------- (Initial assessment) HISTORY - MOI? - Acceleration/Deceleration? - What was it caused by? - Pt restrained? Airbags deployed? Etc. - What are the pt's complaints? - Pt normally wear glasses or contacts? - Pt have hx of eye problems? - Pt ever have eye surgery? - Pt have visual or ocular changes associated with chronic illness? PHYSICAL INSPECTION: - Inspect eye, orbits, face and neck - Check for symmetry, edema, ecchymosis, ptosis, lacerations and hematomas - Inspect globe for lacerations, large corneal abrasions, hyphema, and extrusion or prolapse of intraocular contents - Determine whether lid lac's - Assess pupil's (PERRL) intrathoracic pressure collapses lung on side of injury causing a mediastinal shift that compresses the heart, great vessels, trachea and uninjured lung. Venous return impeded, cardiac output falls, hypotension results. Immediate decompression should be performed. Treatment should not be delayed. What are the S/S of a tension pneumothorax? ---------- Correct Answer --------- - Severe respiratory distress - Markedly diminished or absent breath sounds on affected side - hypotension - Distended neck, head and upper extremity veins-may not be clinically appreciated if significant blood loss present - Tracheal deviation - shift toward uninjured side (LATE sign) - Cyanosis (LATE sign) Define Hemothorax. ---------- Correct Answer --------- Accumulation of blood in the pleural space. What are the S/S of Hemothorax? ---------- Correct Answer --------- - Dyspnea, tachypnea - Chest pain - Signs of shock - Decreased breath sounds on injured side - Dullness to percussion on the injured side What is a pulmonary contusion? ---------- Correct Answer --------- They occur as a result of direct impact, deceleration or high-velocity bullet wounds. It develops when blood leaks into lung parenchyma, causing edema + hemorrhage. This usually develops overtime and not immediately. What are the S/S of pulmonary contusion? ---------- Correct Answer --------- - Dyspnea - Ineffective cough - Hemoptysis - Hypoxia - Chest pain - Chest wall contusion or abrasions What happens to a ruptured diaphragm? ---------- Correct Answer --------- Potentially life- threatening, results from forces that penetrate the body. Left hemidiaphragm is more susceptible to injury because the right side is protected by the liver. - Herniation of abdominal contents - Respiratory compromise b/c impaired lung capacity + displacement of normal tissue. - Mediastinal structures may shift to opposite side of injury What are S/S of a ruptured diaphragm? ---------- Correct Answer --------- (Anything below the nipple line and should be evaluated for potential diaphragmatic injury). - Dyspnea or orthopnea - Dysphagia - Abdominal pain - Sharp epigastric or chest pain radiating to left shoulder (Kehr's sign) - Bowel sounds heard in lower middle chest - Decreased breath sounds on injured side What are S/S with tracheobronchial injury? ---------- Correct Answer --------- Blunt trauma. "Clothesline-type" injuries. - Dyspnea, tachypnea - Hoarseness - Hemoptysis - Subcutaneous emphysema in neck, face, or suprasternal area - Decreased or absent breath sounds - S/S of airway obstruction What are S/S with blunt cardiac injury? ---------- Correct Answer --------- "Cardiac contusion" or "concussion." Common with MVC or falls from heights. - ECG (sinus tach, PVC's, AV blocks) - Chest pain - Chest wall ecchymosis What are the S/S of pericardial tamponade? ---------- Correct Answer --------- A collection of blood in pericardial sac. As blood accumulates, it exerts pressure on the heart, inhibiting or compromising ventricular filling. - Hyotension - Tachycardia or PEA - Dyspnea - Cyanosis - Beck's Triad (hypotension, distended neck veins + muffled heart sounds) - Progressive decreased voltage of conduction complexes on ECG What are aortic injuries S/S? ---------- Correct Answer --------- - Hypotension - Decreased LOC - Hypertension in UE's - Decreased quality (amplitude) of femoral pulses compared to UE pulses - Loud systolic murmur in parascapular region - Chest pain - Chest wall ecchymosis - Widened mediastinum on chest xray - Paraplegia How would you assess a pt with a thoracic injury? ---------- Correct Answer --------- (Initial assessment) Obtain Hx. PHYSICAL: Inspection: - Observe chest wall - Assess breathing effort and RR - Symmetry - Inspect jugular veins (Distended = increased intrathoracic pressure as result of tension pneumothorax or pericardial tamponade. Flat = external jugular veins may reflect hypovolemia) - Inspect upper abdominal region for injury Percussion: - Percuss the chest (Dullness = hemothorax, Hyperresonance = pneumothorax) Palpation: - Palpate chest wall, clavicles and neck for: - Tenderness - Swelling or hematoma - Subcutaneous emphysema - Note presence of bony crepitus - Palpate central and peripheral pulses and compare quality between: - Right and left extremities - Upper and lower extremities - Palpate the trachea (above suprasternal notch. Trach shift may indicate late sign of tension pneumothorax or massive hemothorax) - Palpate extremities for motor and sensory function (lower extremi What is the planning and implementation for thoracic injury? ---------- Correct Answer ---- ----- p. 142 What is kinematics? ---------- Correct Answer --------- A branch of mechanics (energy transfer) that refers to motion and does not consider the concepts of force and mass of the object or body. What is Newton's First Law? ---------- Correct Answer --------- A body at rest will remain at rest. A body in motion will remain in motion until acted on by an outside force. What is the Law of Conservation of Energy? ---------- Correct Answer --------- Energy can neither be created nor destroyed. It is only changed from one form to another. What is Newton's Second Law? ---------- Correct Answer --------- Force equals mass multiplied by acceleration of deceleration. What is kinetic energy (KE)? ---------- Correct Answer --------- KE equals 1/2 the mass (M) multiplied by the velocity squared. What is the Mnemonic for the Initial Assessment? ---------- Correct Answer --------- A = Airway with simultaneous cervical spine protection B = Breathing C = Circulation What is the second thing assessed under the Secondary Assessment? ---------- Correct Answer --------- GIVE COMFORT MEASURES - Talking to pt - Pharmacologic/Nonpharmacologic pain management - Observe for physical signs of pain What is assessed under the Mnemonic "H"? ---------- Correct Answer --------- HISTORY / HEAD-TO-TOE ASSESSMENT - MIVT - M = Mechanism of injury - I = Injuries sustained - V = Vital Signs - T = Treatment - Pt generated information - PMH - Head-to-toe assessment What is assessed under the Mnemonic "I"? ---------- Correct Answer --------- INSPECT POSTERIOR SURFACES - While maintaining C-spine, logroll pt with assistance to inspect back, flanks, buttocks and posterior thighs. - Palpate vertebral column for deformity and areas of tenderness - Assess rectum for presence/absence of tone, presence of blood What she be done after the Secondary Assessment? ---------- Correct Answer --------- Reassess: - Primary survey, - VS - Pain - Any injuries What are factors that contribute to ineffective ventilation? ---------- Correct Answer -------- - - AMS - LOC - Neurologic injury - Spinal Cord Injury - Intracranial Injury - Blunt trauma - Pain caused by rib fractures - Penetrating Trauma - Preexisting hx of respiratory diseases - Increased age What medications are used during intubation? ---------- Correct Answer --------- LOAD Mnemonic: L = Lidocaine O = Opioids A = Atropine D = Defasiculating agents What are the Rapid Sequence Intubation Steps? ---------- Correct Answer --------- PREPARATION: - gather equipment, staffing, etc. PREOXYGENATION: - Use 100% O2 (prevent risk of aspiration). PRETREATMENT: - Decrease S/E's of intubation PARALYSIS WITH INDUCTION: - Pt has LOC, then administer neuromuscular blocking agent PROTECTION AND POSITIONING: - Apply pressure over cricoid cartilage (minimizes likelihood of vomiting and aspiration PLACEMENT WITH PROOF - Each attempt NOT to exceed 30 seconds, max of 3 attempts. Ventilate pt 30-60 seconds between attempts. - After intubation, inflate the cuff - Confirm tube placement w/exhaled CO2 detector. POSTINTUBATION MANAGEMENT: - Secure ET tube - Set ventilator settings - Obtain Chest x-ray - Continue to medicate - Recheck VS and pulse oxtimetry What is a Combitube? ---------- Correct Answer --------- A dual-lumen, dual-cuff airway that can be placed blindly into the esophagus to establish an airway. If inadvertently placed into trachea, it can be used as a temporary ET tube. There are only two sizes: small adult and larger adult. What is a Laryngeal Mask Airway? ---------- Correct Answer --------- Looks like an ET tube but is equipped with an inflatable, elliptical, silicone rubber collar at the distal end. It is designed to cover the supraglottic area. ILMA, does not require laryngoscopy and visualization of the chords. What is Needle Cricothyrotomy ---------- Correct Answer --------- Percutaneous transtracheal ventilation. (temporary) Complications include: - inadequate ventilation causing hypoxia - hematoma formation - esophageal perforation - aspiration - thyroid perforation - subcutaneous emphysema What is Surgical Cricothyrotomy? ---------- Correct Answer --------- Making an incision in cricothyroid membrane and placing a cuffed endo or trach tube into trachea. This is indicated when other methods of airway management have failed and pt cannot be adequately ventilated and oxygenated. Complications include: - Aspiration - Hemorrhage or hematoma formation or both - Lac to trachea or esophagus - Creation of a false passage - Laryngeal stenosis How do you confirm ET Tube/Alternative Airway Placement? ---------- Correct Answer --- ------ - Visualization of the chords - Using bronchoscope to confirm placement - Listening to breath sounds over the epigastrum and chest walls while ventilating the pt - CO2 detector - Esophageal detection device - Chest x-ray How do you inspect the chest for adequate ventilation? ---------- Correct Answer --------- Observe: - mental status - RR and pattern - chest wall symmetry - any injuries - patient's skin color (cyanosis?) - JVD or tracheal deviation? (Tension pneumothorax) What are you looking for when auscultating lung sounds? ---------- Correct Answer ------- -- Absence of BS: - Pneumothorax - Hemothorax - Airway Obstruction Diminished BS: - Splinting or shallow BS may be a result of pain What are you looking for when percussing the chest? ---------- Correct Answer --------- Dullness: - hemothorax Hyperresonance - Pneumothorax Shock stimulates hypothalamus to release corticotropin-releasing hormone that stimulates pituitary to release ACTH that stimulates adrenal gland to release cortisol. Effect of cortisol release is elevation in blood sugar and increased insulin resistance and gluconeogenesis, hepatic process to produce more sugar. Cortisol also causes renal retention of water and sodium, a compensatory mechanism to conserve body water. Explain Hepatic Response. ---------- Correct Answer --------- Liver can store excess glucose as glycogen. As shock progresses, glycogenolysis is activated by epi to break down glycogen into glucose. In a compensatory response to shock, hepatic vessels constrict to redirect blood flow to other vital areas. Explain Pulmonary Response. ---------- Correct Answer --------- Tachypnea happens for 2 reasons: 1. Maintain acid-base balance 2. Maintain increased supply of oxygen * Metabolic acidosis from anaerobic metabolism will be a stimulus for the lungs to increase rate of ventilation. Increased RR is an attempt to correct acidosis + augments oxygen supply to maximize oxygen delivery to alveoli. Explain Irreversible Shock. ---------- Correct Answer --------- Shock uncompensated or irreversible stages will cause compromises to most body systems. - Inadequate venous return - inadequate cardiac filling - decreased coronary artery perfusion - Membranes of lysosomes breakdown within cells and release digestive enzymes that cause intracellular damage. How would you assess someone in hypovolemic shock? ---------- Correct Answer --------- (Use Initial Assessment) and then: Inspect: - LOC - Rate and quality of respirations - External bleeding? - Skin color and moisture - Assess jugular veins and peripheral veins Auscultate: - BP - Pulse pressure - Breath sounds - Heart sounds - Bowel sounds Percuss: - Chest and abdomen Palpate: - Central pulse (carotid or femoral) - Positive inotropic effect (force of contraction) may be evidence by a bounding central pulse - Palpate peripheral pulses - Palpate skin temp and moisture Diagnostic Procedures: - Xrays and other studies - Labs Planning and Implementation - Oxygen - IV's with warmed replacement fluids - Control external bleeding with direct pressure - Elevate LE's - NGT - Foley - Monitor and pulse oximeter - Monitor for development of coagulopathies - Surgery? ICP is a reflection of what three volumes? What happens when one increases? ---------- Correct Answer --------- 1. Brain 2. CSF 3. Blood within the nonexpansible cranial vault As volume of one increases, the volume of another decreases to maintain ICP within normal range. As ICP rises, CPP decreases, leading to cerebral ischemia and potential for hypoxia and lethal secondary insult. Hypotensive pt w/marginally elevated ICP can be harmful. Slightly elevated BP could protect against brain ischemia in a pt with high ICP. Cerebral ischemia can lead to increased concentration of CO2 and decreased concentration of O2 in cerebral vessels. CO2 dilates cerebral blood vessels = increase blood volume and ICP. What are the early signs and symptoms of increased ICP? ---------- Correct Answer ------ --- - Headache - N/V - Amnesia regarding events around the injury - Altered LOC - Restlessness, drowsiness, changes in speech, or loss of judgement What are the late observable signs of symptoms of increased ICP? ---------- Correct Answer --------- - Dilated, nonreactive pupil - Unresponsiveness to verbal or painful stimuli - Abnormal motor posturing patterns - Widening pulse pressure - Increased systolic blood pressure - Changes in RR and pattern - Bradycardia What is Cushing's phenomenon or Cushing's Reflex? ---------- Correct Answer --------- Triad of progressive HTN, bradycardia and diminished respiratory effort. What are the two types of herniation that occurs with ICP? ---------- Correct Answer ------ --- 1. Uncal herniation 2. Central or transtentorial herniation Why does herniation occur? What are the symptoms? ---------- Correct Answer --------- Because of uncontrolled increases in ICP. S/E's - Unilateral or bilateral pupillary dilation - AsyDimmetric pupillary reactivity - Abnormal motor posturing - Other evidence of neurologic deterioration Define uncal herniation. ---------- Correct Answer --------- The uncus (medial aspect of the temporal lobe) is displaced over the tentorium into the posterior fossa. This herniation is the more common of the two types of herniation syndromes. Define central or transtentorial herniation. ---------- Correct Answer --------- A downward movement of the cerebral hemispheres with herniation of the diencephalon and midbrain through the elongated gap of the tentorium. Disruptions of the bony structures of the skull can result in what? ---------- Correct Answer --------- Displaced or nondisplaced fx's causing CSF leakage b/c of lac to the dura mater, creating a passage for CSF. CSF leaks through the nose (rhinorrhea) or the ears (otorrhea). A potential entrance for invading bacteria. Also: meningitis or encephalitis or brain abscess Define Minor Head Trauma. ---------- Correct Answer --------- GCS 13-15 - Incontinence - Sz's What are intracerebral hematoma's and its S/S? ---------- Correct Answer --------- Occur deep within brain tissue, may be single or multiple and commonly associated with contusions (frontal & temporal lobes). They result in significant mass effect, leading to increased ICP and neurologic deterioration. S/S: - Progressive and often rapid decline in LOC - H/A - Signs of increasing ICP - Pupil abnormalities - Contralateral hemiplegia What are the S/S of a linear skull fx? ---------- Correct Answer --------- - H/A - Possible decreased LOC What are the S/S of a depressed skull fx? ---------- Correct Answer --------- - H/A - Possible decreased LOC - Possible open fx - Palpable depression of skull over the fx site What are the S/S of a basilar skull fx? ---------- Correct Answer --------- - H/A - Altered LOC - Periorbital ecchymosis (raccoon eyes), mastoid ecchymosis (Battle's sign), or blood behind tympanic membrane (hemotympanum) - Facial nerve (VII) palsy - CSF rhinorrhea or otorrhea How would you assess a pt with a cranial injury? ---------- Correct Answer --------- (Initial assessment) INSPECTION: - Assess airway - RR, pattern and effort - Assess pupil size and response to light - Unilateral fixed and dilated pupil = oculomotor nerve compression from increased ICP + herniation syndrome - Bilateral fixed and pinpoint pupils indicate a pontine lesion or effects of opiates - Mildly dilated pupil w/sluggish response may be early sign of herniation syndrome - Widely dilated pupil occasionally occurs w/direct trauma to globe of eye - Determine if pt uses eye meds - Abnormal posturing? - Inspect craniofacial area for ecchymosis/contusions - Periorbital ecchymosis - Mastoid's process ecchymosis - Blood behind tympanic membrane - Inspect nose and ears for drainage - Drng present w/out blood, test drng w/chemical reagant strip. Presence of glucose indicated drng of CSF - If drng present and mixed with blood, test by placing drop of fluid on linen or gauze. If a light oute What are signs of a serious eye injury? ---------- Correct Answer --------- - Visual disturbances - Pain - Redness and ecchymosis of the eye - Periorbital ecchymosis - Increased intraocular pressure What is hyphema and its S/S? ---------- Correct Answer --------- Accumulation of blood, mainly RBC's that disperse and layer within the anterior chamber. A severe hymphema obscures entire anterior chamber + will diminish visual acuity severely or completely. Injuries are graded on amount of blood in chamber (Grades I-IV). S/S: - Blood in anterior chamber - Deep, aching pain - Mild to severe diminished visual acuity - Increased intraocular pressure What are s/s of chemical burns to the eye? ---------- Correct Answer --------- Chemical injuries require immediate intervention if it is to be preserved. S/S: - Pain - Corneal Opacification - Coexisting chemical burn and swelling of lids What are S/S of penetrating trauma/open or ruptured globe? ---------- Correct Answer --- ------ - Marked visual impairments - Extrusion of intraocular contents - Flattened or shallow anterior chamber - Subconjunctival hemorrhage, hyphema - Decreased intraocular pressure - Restriction of extraocular movements What are the S/S of orbital fracture (orbital blowout fracture)? ---------- Correct Answer -- ------- - Diplopia (double vision) - Loss of vision - Altered extraocular eye movements - Enophthalmos (displacement of the eye backward into the socket) - Subconjunctival hemorrhage or ecchymosis of the eyelid - Infraorbital pain or loss of sensation - Orbital bony deformity What is LeFort I fracture and its S/S? ---------- Correct Answer --------- Transverse maxillary fx that occurs above level of teeth and results in separation of teeth from rest of maxilla. S/S: - Slight swelling of maxillary area - Possible lip lac's or fractured teeth - Independent movement of the maxilla from rest of face - Malocclusion What is LeFort II fracture and its S/S? ---------- Correct Answer --------- Pyramidal maxillary fx=middle facial area. Apex of fx transverses bridge of nose. Two lateral fx's of pyramid extend through the lacrimal bone of the face and ethmoid bone of skull into the median portion of both orbits. Base of the fx extends above level of the upper teeth into maxilla. CSF leak is possible. S/S: - Massive facial edema - Nasal swelling w/obvious fx of nasal bones - Malocclusion - CSF rhinorrhea What is LeFort III fracture and its S/S? ---------- Correct Answer --------- Complete craniofacial separation involving maxilla, zygoma and bones of cranial base. This fx is frequently associated w/leakage of CSF and fx mandible. S/S: - Massive facial edema - Mobility and depression of zygomatic bones - Ecchymosis - Anesthesia of the cheek - Diplopia - Open bite or malocclusion - CSF rhinorrhea What are the mandibular fracture S/S? ---------- Correct Answer --------- - Malocclusion - Inability to open the mouth (trismus) - Pain, especially on movement - Facial asymmetry and a palpable step-off deformity - Edema or hematoma formation at the fracture site - Blood behind, ruptured, tympanic membrane - Anesthesia of the lower lip What are neck injury S/S? ---------- Correct Answer --------- - Dyspnea - Hemoptysis (coughing up blood) - Subcutaneous emphysema in neck, face, or suprasternal area - Decreased or absent breath sounds - Penetrating wounds or impaled objects Flail segments may not be clinically evident in the first several hours after injury b/c of muscle spasms that cause splinting. After positive pressure intiated, paradoxical chest wall movement ceases. What could a flail chest be associated with? ---------- Correct Answer --------- - Ineffective ventilation - Pulmonary contusion - Lacerated lung parenchyma What are the S/S of flail chest? ---------- Correct Answer --------- - Dyspnea - Chest wall pain - Paradoxical chest wall movement - the flail segment moves in during inspiration and out during expiration. Define Pneumothorax. ---------- Correct Answer --------- Results when an injury to lung leads to accumulation of air in pleural space w/subsequent loss of negative intrapleural pressure. Partial or total collapse of lung may ensue. An open pneumothorax results from wound through chest wall. Air enters pleural space both through the wound and trachea. What are the S/S of a pneumothorax? ---------- Correct Answer --------- - Dyspnea, tachypnea - Tachycardia - Hyerresonance (increased echo produced by percussion over the lung field) on the injured side - Decreased or absent breath sounds on the injured side - Chest pain - Open, sucking wound on inspiration (open pneumothorax) Define tension pneumothorax. ---------- Correct Answer --------- Life-threatening injury. Air enters pleural space on inspiration, but air cannot escape on expiration. Rising intrathoracic pressure collapses lung on side of injury causing a mediastinal shift that compresses the heart, great vessels, trachea and uninjured lung. Venous return impeded, cardiac output falls, hypotension results. Immediate decompression should be performed. Treatment should not be delayed. What are the S/S of a tension pneumothorax? ---------- Correct Answer --------- - Severe respiratory distress - Markedly diminished or absent breath sounds on affected side - hypotension - Distended neck, head and upper extremity veins-may not be clinically appreciated if significant blood loss present - Tracheal deviation - shift toward uninjured side (LATE sign) - Cyanosis (LATE sign) Define Hemothorax. ---------- Correct Answer --------- Accumulation of blood in the pleural space. What are the S/S of Hemothorax? ---------- Correct Answer --------- - Dyspnea, tachypnea - Chest pain - Signs of shock - Decreased breath sounds on injured side - Dullness to percussion on the injured side What is a pulmonary contusion? ---------- Correct Answer --------- They occur as a result of direct impact, deceleration or high-velocity bullet wounds. It develops when blood leaks into lung parenchyma, causing edema + hemorrhage. This usually develops overtime and not immediately. What are the S/S of pulmonary contusion? ---------- Correct Answer --------- - Dyspnea - Ineffective cough - Hemoptysis - Hypoxia - Chest pain - Chest wall contusion or abrasions What happens to a ruptured diaphragm? ---------- Correct Answer --------- Potentially life- threatening, results from forces that penetrate the body. Left hemidiaphragm is more susceptible to injury because the right side is protected by the liver. - Herniation of abdominal contents - Respiratory compromise b/c impaired lung capacity + displacement of normal tissue. - Mediastinal structures may shift to opposite side of injury What are S/S of a ruptured diaphragm? ---------- Correct Answer --------- (Anything below the nipple line and should be evaluated for potential diaphragmatic injury). - Dyspnea or orthopnea - Dysphagia - Abdominal pain - Sharp epigastric or chest pain radiating to left shoulder (Kehr's sign) - Bowel sounds heard in lower middle chest - Decreased breath sounds on injured side What are S/S with tracheobronchial injury? ---------- Correct Answer --------- Blunt trauma. "Clothesline-type" injuries. - Dyspnea, tachypnea - Hoarseness - Hemoptysis - Subcutaneous emphysema in neck, face, or suprasternal area - Decreased or absent breath sounds - S/S of airway obstruction What are S/S with blunt cardiac injury? ---------- Correct Answer --------- "Cardiac contusion" or "concussion." Common with MVC or falls from heights. - ECG (sinus tach, PVC's, AV blocks) - Chest pain - Chest wall ecchymosis What are the S/S of pericardial tamponade? ---------- Correct Answer --------- A collection of blood in pericardial sac. As blood accumulates, it exerts pressure on the heart, inhibiting or compromising ventricular filling. - Hyotension - Tachycardia or PEA - Dyspnea - Cyanosis - Beck's Triad (hypotension, distended neck veins + muffled heart sounds) - Progressive decreased voltage of conduction complexes on ECG What are aortic injuries S/S? ---------- Correct Answer --------- - Hypotension - Decreased LOC - Hypertension in UE's - Decreased quality (amplitude) of femoral pulses compared to UE pulses - Loud systolic murmur in parascapular region - Chest pain - Chest wall ecchymosis - Widened mediastinum on chest xray - Paraplegia How would you assess a pt with a thoracic injury? ---------- Correct Answer --------- (Initial assessment) Obtain Hx. PHYSICAL: Inspection: - Observe chest wall - Assess breathing effort and RR - Symmetry - Inspect jugular veins (Distended = increased intrathoracic pressure as result of tension pneumothorax or pericardial tamponade. Flat = external jugular veins may reflect hypovolemia) - Inspect upper abdominal region for injury Percussion: - Percuss the chest (Dullness = hemothorax, Hyperresonance = pneumothorax) Palpation: - Palpate chest wall, clavicles and neck for: - Tenderness - Swelling or hematoma List common injuries from falls in the older adult population. ---------- Correct Answer ---- ----- What condition is associated with a fall from which the older adult cannot rise? What complications result from this condition? ---------- Correct Answer --------- Review the age-related anatomic and physiologic change of the older adult in relation to the components of the initial assessment. ---------- Correct Answer --------- Describe the fluid resuscitation of an older adult patient related to fluid overload, when to administer red blood cells, and the use of anticoagulant medication. ---------- Correct Answer --------- Describe effects of common medications in relation to the older adult trauma patient. ---- ------ Correct Answer --------- Describe common patterns and severity of injuries in the bariatric trauma patient. --------- - Correct Answer --------- Which comorbid conditions factor into the risks of the bariatric trauma patient? And how? ---------- Correct Answer --------- Describe the pathophysiologic changes of the systems of the bariatric patient and the effects on trauma resuscitation efforts. ---------- Correct Answer --------- Describe techniques to improve the intubation process for the bariatric trauma patient. -- -------- Correct Answer --------- Discuss the use and insertion of nasogastric tubes in the bariatric patient. ---------- Correct Answer --------- Differentiate family and intimate partner violence from community violence. ---------- Correct Answer --------- List the populations at higher risk for interpersonal violence. ---------- Correct Answer ---- ----- Describe the types of abuse and the associated signs of each. ---------- Correct Answer - -------- What cues to abuse may be obtained during the history portion of the initial assessment? ---------- Correct Answer --------- Describe specific injuries associated with interpersonal violence and abuse. ---------- Correct Answer --------- List the basic components of evidence collection. ---------- Correct Answer --------- Describe steps to maintain the forensic chain of custody. ---------- Correct Answer -------- - 1. Preparation and Triage 2. Primary Survery (ABCDE) with resuscitation adjuncts (F,G) 3. Reevaluation (consideration of transfer) 4. Secondary Survey (HI) with reevaluation adjuncts 5. Reevaluation and post resuscitation care 6. Definitive care of transfer to an appropriate trauma nurse ---------- Correct Answer ----- ---- Initial Assessment 1. A- airway and Alertness with simultaneous cervical spinal stabilization 2. B- breathing and Ventilation 3. circulation and control of hemorrhage 4. D - disability (neurologic status) 5. F - full set of vitals and Family presence 6. G - Get resuscitation adjuncts L- Lab results (arterial gases, blood type and crossmatch) M- monitor for continuous cardiac rhythm and rate assessment N- naso or orogastric tube consideration O- oxygenation and ventilation analysis: Pulse oxygemetry and end-tidal caron dioxide (ETC02) monitoring and capnopgraphy H- History and head to toe assessment I- Inspect posterior surfaces ---------- Correct Answer --------- ABCDEFGHI Before the arrival of the pt ---------- Correct Answer --------- When should PPE be placed: Pt is at hospital in the right amount of time, right care, right trauma facility, right resources ---------- Correct Answer --------- Safe Care: Uncontrolled Hemorrhage ---------- Correct Answer --------- Major cause of preventable death: reorganize care to C-ABC ---------- Correct Answer --------- If uncontrolled hemorrhage .. Used at the beginning of the initial assessment 1. A Alert. If the pt is alert he or she will be able to maintain his or her airway once it is clear. 2. V responds to verbal stimuli responds to pain. If the patient needs verbal stimulation to respond, an airway adjunct may be needed to keep the tongue from obstructing the airway. 3. P responds to pain. If the pt. responds only to pain, he or she may not be able to maintain his or her airway adjunct may need to be placed while further assessment is made to determine the need for intubation. 4. U Unresponsive. If the pt. is unresponsive, announce it loudly to the team and direct someone to chk in the pt is pulseless while assessing if the cause of the problem is the airway. ---------- Correct Answer --------- Airway and AVPU: ask pt to pen his or her mouth ---------- Correct Answer --------- While assessing airway the patient is alert and responds to verbal stimuli you should.. jaw thrust maneuver to open airway and assess for obstruction. If pt has a suspected csi, the jaw thrust procedure should be done by two providers. One provider can maintain c-spine and the other can perform the jaw thrust maneuver. ---------- Correct Answer --------- While assessing airway pt is unable to open mouth, responds only to pain, or is unresponsive you should.. 1. The tongue obstructing the airway 2. loose or missing teeth 3. foreign objects 4. blood, vomit, or secretions' 5. edema 6. burns or evidence of inhalation injury Auscultiate or listen for: 1. Obstructive airway sounds such as snoring or gurgling 2. Possible occlusive maxillofacial bony deformity 3. Subcutaneous emphysema ---------- Correct Answer --------- Inspect the mouth for: 1. Check the presence of adequate rise and fall of the chest with assisted ventilation 2. Absence of gurgling on auscultation over the epigastrium 3. Bilateral breath sounds present on auscultation 4. Presence of carbon dioxide (CO2) verified by a CO2 device or monitor ---------- Correct Answer --------- If the pt has a definitive airway in what should you do? 1. Suction the airway 2, Use care to avoid stimulating the gag reflex 3. If the airway is obstructed by blood or vomitus secretions, use a rigid suction device If foreign body is noted, remove it carefully with forceps or another appropriate method - --------- Correct Answer --------- If Airway is not patent 1. Apnea 2. GCS 8 or less 3. Maxillary fractures Reevaluation and Consider the need to Transfer ---------- Correct Answer --------- Final step in primary survey H,I ---------- Correct Answer --------- Secondary Survery History and Head to toe MIST - prehospital report MOI Injuries sustained S s/s in the field T treatment in the field if patients family present get a better hx on them ---------- Correct Answer --------- H Sample is part of history S symptoms associated with injury A allergies and tetanus status M meds currently on including anticoagulant therapy P past medical hx L last oral intake E Events and environment factors related to the injury ---------- Correct Answer --------- SAMPLE inspect for lacs, abrasions, asymmetry of facial expressions palate for depressions and tenderness look at ears for drainage ---------- Correct Answer --------- Head to toe assessment: Head and face immobilize cervical spine, tenderness, tracheal deviation ---------- Correct Answer -------- - Head to toe assessment: Neck and cervical spine inspect, auscultate, palpate any spontaneous breathing, rate, depth, and degree of effort, use of accessory muscles lacs, contusions, auscilate lung sounds and heart sounds ---------- Correct Answer --------- Head to toe assessment: Chest don't forget flanks!!! inspect of lacs, puncture wounds, contusions, auscultate then palpate: bowel sounds? any rigidity, guarding? begin with light palpation start to palpate with side that does not hurt maybe do a fast scan? ---------- Correct Answer --------- Head to toe assessment: Abdomen any lacs? deformities? blood at the urtheral meatus palpate pelvis with high pressure over the iliac wings downward and medially ---------- Correct Answer --------- Head to toe assessment: pelvis and perineum any deformities? bleeding? contusions, lacs? skin temp?? place splints on deformities, pulses ---------- Correct Answer --------- Head to toe assessment: Extremities inspect posterior surfaces blogroll with at least 3 people. maintain c spine take out backboard Rectal tone per MD ---------- Correct Answer --------- I labs, wound care, tetanus, administer meds, prepare for transfer ---------- Correct Answer --------- Secondary Reval Adjuncts Vital signs Interventions Primary survey Pain ---------- Correct Answer --------- Post resuscitation care parameters that are continuously evaluated: Capnography monitors numeric value, as well as continuous waveform, indicating real- time measurement and trending over time. ---------- Correct Answer --------- Quantitative: Colorimetric CO2 detectors provide info about the presence or absence of CO2. A chemically treated indicator strip changes color revealing the presence or absence of exhaled CO2 ---------- Correct Answer --------- Qualitative D displaced tube O obstructed or kinked P pneumothorax E equipment failure , such as becoming detached from the equipment or loss of capnopgrahy ---------- Correct Answer --------- DOPE 1. Preparation 2. Preoxygenation 3. Pretreatment 4. Paralysis and Induction 5. Protecting and positioning - v 6. Placement of proof - secure the tube 7. Post intubation - secure ETT Tube, get X-ray for placement ---------- Correct Answer - -------- Steps of Rapid Sequence Intubation from hemorrhage is leading cause. Hypovolemia is caused by decrease in the amount of circulating volume. Goal is to replace volume. ---------- Correct Answer --------- Hypovolemic Shock results from hypo perfusion to the tissue due to an obstruction in either vasculature or heart. Goal is to relieve obstruction and improve perfusion. Ex: tension pneumo or cardiac tamponade are two classic examples that may result from trauma. ---------- Correct Answer --------- Obstructive Shock Results from pump failure in the presence of adequate intravascular volume. There is a lack of cardiac output and end organ perfusion secondary to a decrease in myocardial contractility and/or valvular insufficiency. Ex: MI's or dysrhythmia are common causes ---------- Correct Answer --------- Cariogenic Shock occurs as a result of maldistribution of an adequate circulating blood volume with the loss of vascular tone or increased permeability. Ex: Anaphylactic - release of antihistamines Septic Shock - systemic release of bacterial endotoxins, resulting in increased vascular permeability and vasodilation. Neurogenic shock - spinal cord injury results of loss in sympathetic nervous system control of vascular tone. Goal: Volume replacement and vasoconstriction ---------- Correct Answer --------- Distributive Shock A breath every 5 to 6 seconds: 10-12 ventilations per minute ---------- Correct Answer --- ------ Bag mask ventilation Stroke Volume X HR ---------- Correct Answer --------- Cardiac Output = What are the greatest risks for transport? ---------- Correct Answer --------- Loss of airway patency, displaced obstructive tubes lines or catheters, dislodge splinting devices, need to replace or reinforce dressings, deterioration in patient status change in vital signs or level of consciousness, injury to the patient and/or team members According to newtons law which of these two force is greater: size or force? ---------- Correct Answer --------- Neither. For each force there is an equal and opposite reaction. What is the relationship between mass and velocity to kinetic energy? ---------- Correct Answer --------- Kinetic energy is equal to 1/2 the mass multiplied the square of its velocity therefore when mass is doubled so is the net energy, however, when velocity is doubled energy is quadrupled. What is tension? ---------- Correct Answer --------- stretching force by pulling at opposite ends What is compression? ---------- Correct Answer --------- Crushing by squeezing together What is bending? ---------- Correct Answer --------- Loading about an axis. Bending causes compression on the side the person is bending toward intention to the opposite side What is shearing? ---------- Correct Answer --------- Damage by tearing or bending by exerting faucet different parts in opposite directions at the same time. What is torsion? ---------- Correct Answer --------- Torsion forces twist ends in opposite directions. What is combined loading? ---------- Correct Answer --------- Any combination of tension compression torsion bending and/or shear. What are the four types of trauma related injuries? ---------- Correct Answer --------- Blunt, penetrating, thermal, or blast. What are contributing factors to injuries related to blunt traumas? ---------- Correct Answer --------- The point of impact on the patient's body, the type of surface that is hit, the tissues ability to resist (bone versus soft tissue, air-filled versus solid organs), and the trajectory of force. What are the seven patterns of pathway injuries related to motor vehicle accidents? ----- ----- Correct Answer --------- Up and over, down and under, lateral, rotational, rear, roll over, and ejection. Differentiate between the three impacts of motor vehicle impact sequence. ---------- Correct Answer --------- The first impact occurs when the vehicle collided with another object. The second impact occurs after the initial impact when the occupant continues to move in the original direction of travel until they collide with the interior of the vehicle or meet resistance. The third impact occurs when internal structures collide within the body cavity. What are the three factors that contribute to the damage caused by penetrating trauma's? ---------- Correct Answer --------- The point of impact, the velocity and speed of impact, and the proximity to the object. What causes the primary effects of blast traumas? ---------- Correct Answer --------- The direct blast effects. Types of injuries include last long, tympanic membrane rupture and middle ear damage, abdominal hemorrhage and perforation, global rupture, mild Trumatic brain injury. What causes the secondary effects of blast traumas? ---------- Correct Answer --------- Projectiles propelled by the explosion. Injuries include penetrating or blunt injuries or I penetration. What causes the tertiary effects of blast traumas? ---------- Correct Answer --------- Results from individuals being thrown by the blast wind. Injuries include hole or partial body translocation from being thrown against a hard service: blunt or penetrating trauma's, fractures, traumatic amputations. What causes quarternary effects of blast traumas? ---------- Correct Answer --------- All explosion related injuries, illnesses, or diseases not due to the first three mechanisms. Injuries include external and internal burns, crush injuries, closed and open brain injuries, asthmatic or breathing problems from dust smoke or toxic fumes, angina, or hyper glycemia and hypertension. What causes quinary effects of blasts traumas? ---------- Correct Answer --------- Those associated with exposure to hazardous materials from radioactive, biologic, or chemical components of a blast. Injuries include a variety of health effects depending on agent. What are the three processes that transfer oxygen from the air to the lungs and blood stream ---------- Correct Answer --------- Ventilation: the active mechanical movement of air into and out of the lungs; diffusion: the passive movement of gases from an area of higher concentration to an area of lower concentration; and perfusion: the movement of blood to and from the lungs as a delivery medium of oxygen to the entire body. When would you use a nasopharyngeal airway versus an oral pharyngeal airway? ------- --- Correct Answer --------- Nasopharyngeal airways is contraindicated in patients with facial trauma or a suspected basilar skull fracture. Oral pharyngeal airways is used in unresponsive patients unable to maintain their airway, without a gag reflex as a temporary measure to facilitate ventilation with a bag mask device or spontaneous ventilation until the patient can be intubated. Describe the measurement of an NPA ---------- Correct Answer --------- Measure from the tip of the patient's nose to the tip of the patients earlobe. Measurement of an OPA ---------- Correct Answer --------- Place the proximal end or flange of the airway adjunct at the corner of the mouth to the tip of the mandibular angle. True or false: NPAs and OPAs are definitive airways. ---------- Correct Answer --------- False. When placing one of these? One should consider the potential need for a definitive airway. Name the three ways to confirm ETT placement ---------- Correct Answer --------- Placement of a CO2 monitoring device, Assessing for equal chest rise and fall, and listening at the epigastrium and four lung fields for equal breath sounds. When capnography measurement reads greater than 45MMHG, the nurse should consider increasing or decreasing the ventilation rate? ---------- Correct Answer --------- Increasing the ventilation rate. Doing so would allow the patient to blow off retained CO2. When capnography measurement reads less than 35MMHG, the nurse should consider increasing or decreasing the ventilation rate? ---------- Correct Answer --------- Decreasing the ventilation rate. By doing so, the nurse allows the patient to retain CO2. What are the three stages of shock ---------- Correct Answer --------- Compensated, decompensated or progressive, and irreversible. What are the signs of compensated shock? ---------- Correct Answer --------- Anxiety, confusion, restlessness, increased respiratory rate, narrowing pulse pressure were diastolic increases yet systolic remains unchanged, tachycardia with bounding pulses, and decreased urinary output What are the signs and symptoms of decompensated shock? ---------- Correct Answer -- ------- Decreased level of consciousness, hypertension, narrow pulse pressure, tachycardia with weak pulses, tachypnea, skin that is cool clammy and cyanotic, base access outside the normal range, and serum lactate levels greater than two to 4MMOL/L. What are the signs and symptoms of irreversible shock? ---------- Correct Answer --------- Obtunded stuporous or comatose state, marked hypertension and heart failure, bradycardia with possible dysrhythmias, decreased and shallow respiratory rate, pale cool and clammy skin, kidney liver and other organ failure, severe acidosis, elevated lactic acid levels, worsening base access on ABGs, coagulopathies with petechiae purpura or bleeding. What are the treatment goals for a TBI? ---------- Correct Answer --------- O2 saturation > or equal to 95%, systolic blood pressure > or equal to 100 MMHG, ICP < 15 MMHG, CPP > or equal to 60 MMHG, normal glycemia, hemoglobin > or equal to 7 g/DL, sodium 135-145, osmotic diuretics, anti-emetics, sedatives, anticonvulsants, head of bed at 30°, and neck at midline A 56 y/o M involved in a MVC is brought to the ED. He complains of neck pain, SOB, and diffuse abdominal pain. His GCS is 15, VS: BP 98/71, HR 125, RR 26, SpO2 94% on high-flow O2 via NRB mask. Which is the priority intervention for this patient? --------- - Correct Answer --------- Expedite transfer to the closest trauma center Which of the following considerations is the most important when caring for a geriatric trauma pt? ---------- Correct Answer --------- a pertinent medical hx is crucial Following a review of recent drills and a real disaster event, a hospital has identified deficiencies and is taking steps to minimize the impact of a future disaster . Which phase of the disaster life cycle does this describe? ---------- Correct Answer --------- Mitigation EMS brings a pt who fell while riding his bicycle. Using the American College of Surgeobs screening guidelines, which assessment finding would prompt the RN to prepare the pt for a radiologic spine clearance? ---------- Correct Answer --------- smell of alcohol on breath What is the effect of hypothermia on the oxyhemoglobin dissociation curve? ---------- Correct Answer --------- hemoglobin does not readily release O2 for use by the tissues Which of the following is a component of the trauma triad of death? ---------- Correct Answer --------- acidosis EMS brings a pt from MVC. VS: BP 90/49, HR 48, RR 12, temp 97.2F (36.2 C). The pt exhibits urinary incontinence and priapism. These assessment findings are most consistent with which of the following types of spinal cord injury? ---------- Correct Answer --------- Complete Which of the following is an expected finding in a pt with a tube thoracstomy connected to a chest drainage system? ---------- Correct Answer --------- flucuation in the water seal chamber During the primary survey of an unconscious pt with multi-system trauma, the nurse notes snoring respirations. What priority nursing interventions should be preformed next? ---------- Correct Answer --------- insert an oropharyngeal airway if there is no gag reflex A 35 y/o M presents with facial trauma after being struck in the face with a baseball. A teardrop-shaped left pupil is noted on exam. What type of injury is suspected? ---------- Correct Answer --------- globe rupture A trauma pt is restless and repeatedly asking "where am i?" VS upon arrival: BP 110/60, HR96, RR 24. Her skin is cool and dry. Current VS are BP 104/84, HR 108, RR 28. The pt is demonstrating s/sx of which stage of shock? ---------- Correct Answer ------- -- compensated An unresponsive trauma pt has an oropharygeal airway in place, shallow and labored respirations, and dusky skin. The trauma team has administered medications for drug- assisted intubation and attempted intubation but was unsuccessful. What is the most appropriate immediate next step? ---------- Correct Answer --------- ventilate with a bag mask device When is the tertiary survey completed fora trauma pt? ---------- Correct Answer --------- within 24 hrs of trauma An intubated and sedated pt in the ED has multiple extremity injuries with the potential for causing compartment syndrome. What is the most reliable indication of compartment syndrome in a patient who is unconscious? ---------- Correct Answer --------- pressure Which of the following is possible complication of positive-pressure ventilation? ---------- Correct Answer --------- worsening pneumothorax the most reassuring finding for a male pt with hip pain after a fall is which of the following? ---------- Correct Answer --------- pelvic stability Which of the following pulse pressures indicate early hypovolemic shock? ---------- Correct Answer --------- narrowed Patients with a crush injury should be monitored for which of the following conditions? -- -------- Correct Answer --------- dysrhythmias Tearing of the bridging veins is most frequently associated with which brain injury? ------ ---- Correct Answer --------- subdural hematoma A 20 y/o M presents to the ED complaining of severe lower abd pain after landing hard on the bicycle cross bars while preforming an aerial BMX maneuver. Secondary assessment reveals lower abd tenderness and scrotal ecchymosis. Which of the following orders would the RN question? ---------- Correct Answer --------- straight cath for urine sample You are caring for a pt who was involved in a MVC and is 32 weeks pregnant. Findings of your secondary survey include abd pain on palpation, fundal ht at the costal margin, and some dark bloody show. Varying accelerations and decelerations are noted on cariocgraphy. These findings are most consistent with which of the following? ---------- Correct Answer --------- placental abruption Which of the following is true about the log-roll? ---------- Correct Answer --------- it can worsen cord damage from an unstable spinal injury All of these are considered a critical communication point in trauma care EXCEPT which of the following? ---------- Correct Answer --------- defusings What bedside monitoring parameters are used to assess for adequacy of O2 and effectiveness of ventilation? ---------- Correct Answer --------- pulse oximetry and capnography Caregivers carry in a 2 y/o into the ED who fell out of a second-story window. The pt is awake and crying with increased work of breathing and pale skin. Which of the following interventions has the highest priority? ---------- Correct Answer --------- padding the upper back while stabilizing the cervical spine Which of the following injuries is LEAST likely to be promptly identified? ---------- Correct Answer --------- bowel (Hollow organ injury signs and symptoms may be subtle and delayed but are associated with a high mortality rate if not identified.) A pt is brought to the ED of a rural hospital following a high-speed MVC. When significant abd and pelvic injuries are noted in the primary survey, which of the following is the priority interventions? ---------- Correct Answer --------- Initiate transfer to a trauma center A pt with a complete spinal cord injury in neurogenic shock will demonstrate hypotension and which other clinical signs? ---------- Correct Answer --------- bardycardia and absent motor function below the level of injury a 37 y/o F has a deformity of the L wrist after a fall. She is reluctant to move her hand due to pain. Which of the following is the most appropriate intervention? ---------- Correct Answer --------- apply splint and elevate above the level of the heart which of the following occurs during the third impact of a motor vehicle crash? ---------- Correct Answer --------- the aorta is torn at its attachment with the ligamentum arteriosum a 5 y/o child presents to the ED with bruises to the upper arm and buttocks in various stages of healing and multiple small, clean, round burns to the back. There are no abnormalities found based on the pediatric assessment triangle or primary survey. Which of the following is the priority survey. Which of the following is the priority nursing intervention? ---------- Correct Answer --------- Report your suspicion of maltreatment in accordance with local regulations What finding raises suspicion of a complete spinal cord injury? ---------- Correct Answer - -------- Priapism A trauma patient is en route to a rural emergency department. Radiology notifies the charge nurse that the computerized tomography (CT) scanner will be out of service for several hours. The team gathers to plan accordingly. Which of the following terms best describes this trauma teams communication? ---------- Correct Answer --------- huddle? An intubated trauma patient is being transferred to a tertiary care center. After moving the patient to the stretcher for transport, a drop in pulse oximetry to 85% is noted. Which of the following is the priority intervention? ---------- Correct Answer --------- Confirm ETT placement The nurse is caring for a 120 kg male brought in after a warehouse fire and is calculating the patients fluid resuscitation needs. He has painful red blistering to the entire surface of both upper extremities and superficial burns to the anterior chest. Using the modified Lund and Browder chart to calculate the total body surface area burned, how much IV fluid would be administered in the first 8 hours? ---------- Correct Answer --------- 2280 ml (Both upper arms = 8%, both lower arms = 6%, and both hands = 5%, yielding 19 * 2 * 120 = 4560 mL. Half of the total is equal to 2280 mL. Superficial burns are not included in the TBSA calculation for fluid resuscitation.) Associated with significant geriatric morbidity and mortality. ---------- Correct Answer ----- ---- Which of is TRUE about rib fractures? -Immediate treatment is 3-sided dressing -Definitive treatment is Chest Tube ---------- Correct Answer --------- For Open Pneumothorax treatments are - will cause obstructive shock - If Chest Tube placement is not ready available, needs Needle Decompression with 14- gauge needle, longer is better over the 2nd Intercostal space, midclavicular line OR 5th intercostal space, anterior axillary line ---------- Correct Answer --------- During Tension Pneumothorax -Decrease/Absent breath sounds ipsilaterally -JVD -Hypotension -Hyper-resonance ipsilaterally -Tracheal Deviation (late sign) -Severe Respiratory distress, Anxiety, agitation -Can lead to PEA ---------- Correct Answer --------- S/S of Tension Pneumothorax -Persistent Blood lost following chest tube insertion -Hemorrhagic shock: pallor, tachycardia, hypotension, cool peripheries -External thoracic injury -Decrease Breath sounds and decrease chest movement - dullness ipsilaterally -Anxiety/ Agitation. ---------- Correct Answer --------- Hemothorax: Blood in the pleural space. Assessment Finding What is the best measure of the adequacy of cellular perfusion and helps predict the outcome of resuscitation? ---------- Correct Answer --------- Base deficit used in conjunction with serum lactate Will hypocapnia cause vasoconstriction or vasodilation, especially in the cerebral vasculature? ---------- Correct Answer --------- Vasoconstriction What results from tissue hypo perfusion and oxygen deficit? ---------- Correct Answer ---- ----- Metabolic acidosis What type of shock results in generalized vasodilation? ---------- Correct Answer --------- Neurogenic shock Spinal cord injuries at C3-C5 causes loss of what nerves function, resulting in what? ---- ------ Correct Answer --------- Phrenic nerve; paralyzed diaphragm and inability to breath Extremity elevation AT the level of the heart is beneficial for what type of injury? ---------- Correct Answer --------- Compartment Syndrome What is a high risk of frostbite? ---------- Correct Answer --------- Thrombus formation What two medications can be administered to maintain perfusion after a frostbite injury along with rewarming? ---------- Correct Answer --------- Tissue plasminogen activator or non steroidal anti-inflammatory medication (NSAIDS) An rise in diastolic blood pressures is a sign of increasing what? ---------- Correct Answer --------- Peripheral resistance What position will benefit the airway and work of breathing for the bariatric patient? ------ ---- Correct Answer --------- Reverse Trendelenburg Which phase of a blast results from flying debris, projectiles, and bomb fragments causing lacerations or penetrating injuries? ---------- Correct Answer --------- Secondary Phase What phase of a blast results from any explosion-related illness or injury including hyperglycemia, hypertension, angina, asthma, COPD, or sepsis? ---------- Correct Answer --------- Quaternary Phase What phase of a blast results from individuals being thrown by the blast and impacting walls, ground, or any hard object? ---------- Correct Answer --------- Tertiary Phase What phase of a blast results from impact of the over and under pressurization wave with body surfaces. Injuries include blast lung, tympanic membrane rupture, abdominal hemorrhage, globe rupture, and mild traumatic brain injury? ---------- Correct Answer ---- ----- Primary Phase Signs of what include muscle pain or weakness, dark red or brown urine, general weakness or malaise, and elevated creatinine kinase levels? ---------- Correct Answer --- ------ Rhabdomyolosis Signs of what include anxiety, pleuritic chest pain, dyspnea, hypoxemia, hemoptysis, cough, orthopnea, adventitious lung sounds, decreased lung sounds, jugular vein distention, or hypotension? ---------- Correct Answer --------- Pulmonary Embolus Signs of what include headache, nausea and vomiting, amnesia, behavioral changes, altered level of consciousness? ---------- Correct Answer --------- Increased intracranial pressure Signs of what include asymmetric pupillary reactivity, unilateral dilation, widening pulse pressure, abnormal motor posturing, bradycardia, and decreased respiratory effort? ----- ----- Correct Answer --------- Late signs of increased ICP with Herniation Syndrome What is caused by the tear of the bridging veins or middle meningeal artery? ---------- Correct Answer --------- Subdural and Epidural Hematoma Affect concentration, memory, sleep, mode, and libido. Causes headaches, dizziness and nausea. ---------- Correct Answer --------- Postconcussive Syndrome/ Mild Traumatic Brain Injury Signs and symptoms similar to early signs of increased ICP but do not worsen. ---------- Correct Answer --------- Postconcussive Syndrome/ Mild Traumatic Brain Injury Cerebral Perfusion Pressure = ______-______ ---------- Correct Answer --------- MAP - ICP What is the range for CO2 to maximize perfusion? ---------- Correct Answer --------- 35- 45 Does hypoventilation cause dilation or constriction? Increase or decrease ICP? ---------- Correct Answer --------- Dilation and increase in ICP due to high CO2 Hyperventilation cause dilation or constriction? ---------- Correct Answer --------- Constriction d/t low CO2 Pupils with pressure on cranial nerve ---------- Correct Answer --------- Fixed and dilated Wet, waxy, red to pale skin that does not blanch, multiple sisters. What degree burn? --- ------- Correct Answer --------- Second degree; deep partial thickness Waxy white to leathery gray to charred skin that is dry and firm with absent hair, no blanching. What degree burn? ---------- Correct Answer --------- Third degree; full thickness Burn extends into the fascia and/or muscle. What degree burn? ---------- Correct Answer --------- Fourth degree; full thickness Zones of burn injury (3) ---------- Correct Answer --------- Zone of coagulation Zone of stasis Zone of hyperemia What syndrome occurs most often in burns greater than 20% BSA and lasts for approximately 6-12 hours? ---------- Correct Answer --------- Capillary Leak Syndrome Fluid replacement goals/calculation ---------- Correct Answer --------- 2mL/kg LR x TBSA% (give 1/2 over first 8 hours (minus transport time) and 1/2 over next 16 hours) Urine Output goals for fluid replacement therapy ---------- Correct Answer --------- 0.5 mL/kg (30-50mL/hr) Disaster Definition ---------- Correct Answer --------- A sudden calamitous event that seriously disrupts the functioning of a community or society and causes human, material, and economic losses that exceed the community's or society's ability to cope using its own resources. Mitigation ---------- Correct Answer --------- A foundation to limit the potential impact of a disaster Triage colors and meanings: ---------- Correct Answer --------- Green - minor, walking wounded Yellow - delayed Red - immediate Black - expectant, deceased When should the decision to transfer be made? ---------- Correct Answer --------- When the patient's needs outweigh the capabilities of your facility Tertiary care facility, teaching hospital, comprehensive care from resuscitation or rehabilitation, research, injury prevention. ---------- Correct Answer --------- Level 1 Trauma Center Trauma rescucitation and definitive trauma care. Specialty and rehabilitation care may not be as comprehensive, may not conduct research. ---------- Correct Answer --------- Level 2 Trauma Center Provides ATLS assessment, intervention, resuscitation and emergency surgery. Generally does not accept transfers. ---------- Correct Answer --------- Level 3 Trauma Center Provides ATLS assessment, intervention, and resuscitation. May be in a remote area, has 24 hour physician and NP coverage. ---------- Correct Answer --------- Level 4 Trauma Center Greatest risk to the patient during interfaculty transfer/transport? ---------- Correct Answer --------- Loss of airway and respiratory compromise What survey is a complete exam within 24 hours of arrival and identified injuries missed during primary assessment, reviews radiology studies, orders more studies, and assesses for hidden injuries? ---------- Correct Answer --------- Tertiary Survey Stages of shock ---------- Correct Answer --------- 1. Compensatory Shock 2. Decompensatory Shock (progressive, hypotensive) 3. Irreversible Shock Anxiety, confusion, restlessness, narrowing pulse pressure, tachycardia, bounding pulse, and decreasing urinary output are signs of what? ---------- Correct Answer --------- Compensated Shock Decreased level of consciousness, hypotension, narrowed pulse pressure, tachycardia with weak pulse, tachypnea, and cool, clammy, cyanotic skin are signs of what? ---------- Correct Answer --------- Decompensated/Progressive shock Obtunded/comatose, profound hypotension, bradycardia, dysrhythmias, slow shallow respirations, petechiae/purpura are signs of what? ---------- Correct Answer --------- Irreversible shock Benefits of the trauma nursing process ---------- Correct Answer --------- Systematic approach to the evaluation of each trauma patient. Identifies life-threatening conditions, determines priorities of care. Efficient production of ATP, which maintains cellular metabolic function, is seen with what type of metabolism? ---------- Correct Answer --------- Aerobic metabolism Inefficient production of ATP, byproduct is lactic acid, leads to metabolic acidosis, cellular dysfunction leads to cell death with what type of metabolism? ---------- Correct Answer --------- Anaerobic metabolism The cellular process in which oxygen is used to metabolize glucose. Energy is produced in an efficient manner with minimal waste products. ---------- Correct Answer --------- Aerobic metabolism The cellular process in which glucose is metabolized into energy without oxygen. Energy is produced in an inefficient manner with many waste products. ---------- Correct Answer --------- Anaerobic metabolism What is included in the Trauma Triad of Death? ---------- Correct Answer --------- Coagulopathy Acidosis Hypothermia Pump failure, caused by blunt cardiac injury, dysrhythmias, or myocardial infarction is what type of shock? ---------- Correct Answer --------- Cardiogenic Shock What type of shock is caused by cardiac tamponade or tension pneumothorax? Ventricular failure is seen. ---------- Correct Answer --------- Obstructive Shock Reservoir depletion and hemorrhage cause which type of shock? ---------- Correct Answer --------- Hypovolemic Shock Vasodilation, anaphylaxis, sepsis, and spinal cord injuries cause which type of shock? -- -------- Correct Answer --------- Distributive Shock Options for hemorrhage ---------- Correct Answer --------- Pressure at site, tourniquet, hemostatic dressings, Massive Transfusion Protocol, Tranexamic Acid (clotting promoter) Treatment for cariogenic shock ---------- Correct Answer --------- inotropic support, anti- dysrhythmic medications, treat myocardial infarction or other underlying cause Treatment for obstructive shock ---------- Correct Answer --------- pericardiocentesis, cardiac window, needle decompression, position pregnant patient on L side Treatment for distributive shock ---------- Correct Answer --------- Support ventilations, vasopressors, pain management, apply warming methods Treatment for hypovolemic shock ---------- Correct Answer --------- Tourniquet, 1:1:1 blood products, massive transfusion protocol, TXA, surgical repair Subdural hematoma is caused by tearing of the ______ veins and symptoms usually present within ____ hours of the accident. ---------- Correct Answer --------- Bridging; 72 Epidural hematoma is caused by an arterial or venous bleed? Sx are transient LOC followed by a lucid period. ---------- Correct Answer --------- Arterial Answer: A ---------- Correct Answer --------- A) report your suspicion of the maltreatment in accordance with local regulations *** B) apply ice to the bruises and consult wound care C) engage in therapeutic communication to determine the MOI D) provide the family with injury prevention resourse A patient with lower extremity fracture complains of severe pain and tightness in his calf, minimally relieved by pain medications. Which of the following is the priority nursing intervention? Answer: C ---------- Correct Answer --------- A) elevating the extremity above the level of the heart B) repositioning and apply ice C) Elevating the extremity to the level of the heart *** D) Preparing the patient for ultrasound A 36-year old female has a deformity of the left wrist after a fall. She is reluctant to move her hand due to pain. Which of the following is the most appropriate intervention? Answer: B ---------- Correct Answer --------- A) apply a sling and elevate the extremity to the level of the heart. B) apply a splint and ELEVATE ABOVE THE LEVEL OF THE HEART *** C) Apply a sling and elevate the extremity above the level of the heart D) apply a splint and elevate the extremity to the level of the heart. Which of the following is a late sign of increased intracranial pressure? Answer: C Cushing Response Widening pulse pressure Reflex bradycardia Decreased RR ---------- Correct Answer --------- A) Restlessness or drowsiness B) Nausea and vomiting C) Decreased respiratory effort** D) amnesia and anxiety Which of the following mnemonics can help the nurse prioritize care for a trauma patient with massive uncontrolled hemorrhage? Answer: B ---------- Correct Answer --------- A) ABC B) MARCH*** C) AVPU D) VIPP A patient is thrown against a car during a tornado and presents with obvious bilateral femoral fractures. The patient is pale, alert, disoriented, and has delayed cap refill. Which of the following interventions would be most appropriate for this patient based on the disaster triage principle? Answer: A ---------- Correct Answer --------- A) initiate two large caliber intravenous lines for Ringers lactate solution administration *** B) Administer Dilaudid for pain control and provide comfort care C) Place the patient in an observation area for care within the next few hours D) Contact the command center for the personnel to notify next of kin. A patient arrives with a large open chest wound after being assault ed with a machete, Prehospital providers placed a nonporous dressing over the chest wound and tapes it on 3 sides. He is now showing signs of anxiety, restlessness, severe respiratory distress, cyanosis, and decreasing blood pressure. Which of the following is the MOST appropriate interventions? Answer: C ---------- Correct Answer --------- A) needle decompression B) tube thoracostomy C) dressing removal ** D) surgical repair A patient is found lying on the floor after falling 13 hours ago. Which of the following lab values is expected with a musculoskeletal complication associated with this presentation? Answer: A ---------- Correct Answer --------- A) elevated creatine kinase *** B) decreased potassium level C) decreased WBC D) elevated GFR A 56-year- male patient involved in a MVC is brought to the ED of a rural critical access facility. He c/o neck pain, SOB, and diffuse abd pain. His GCS is 15. His vitals are as follows: BP 98/71 HR 125 beats/min RR 26 breaths/min SpO2 94% on high flow O2 via NRB mask which of the following is the priority intervention for this patient? Answer: C ---------- Correct Answer --------- A) Expedite transport to the CT scanner B) prepare the patient for spinal radiographs C) expedite transfer to the closest trauma center *** D) notify the patients family Caregivers carry a 2-year old into the ED who fell out of 2nd story window. The patient is awake and crying with increased work of breathing and pale skin. which of the following interventions has the highest priority? Answer: A ---------- Correct Answer --------- A) padding the upper back while stabilizing the cervical spine *** B) applying a tight-fitting NRB mask with an attached resevior C) establishing intravenous access and administering a 20mL/kg bolus D) preparing for drug assisted intubation Which of the following is a component of the trauma triad of death? Answer: A hypothermia, metabolic acidosis, coagulopathy ---------- Correct Answer --------- A) Acidosis ** B) hyperthermia C) hemorrhage D) sepsis A 35-year old male presents with facial trauma after bring struck inthe face with a baseball. a tear-drop shaped left pupil is noted on exam. What type of injury is suspected? Answer: B ---------- Correct Answer --------- A) oculomotor nerve palsy B) globe rupture *** C) uncal herniation D) retinal detachment Which of the following is true about the log roll maneuver? Answer: C ---------- Correct Answer --------- A) it causes less spinal motion than the lift and slide manuever B) it is recommended for patients with unstable pelvic fractures C) it can worsen cord damage from an unstable spinal injury *** D) it does not increases the risk of life threatening hemorrhage from unstable injuries EMS arrives with the intoxicated driver of a car involved in a MVC. EMS reports significant damage to the drivers side of the car. The patient is asking to have the cervical color removed. when is it appropriate to remove the cervical collar? Answer: A ---------- Correct Answer --------- A) after a physical examination if the patient has no radiologic abnormalities on a computed tomography scan *** B) after physician evaluation if the patient has not neurologic abnormalities on exam C) after palpation of the spine if the patient has no point tenderness the the vertebral column
Docsity logo



Copyright © 2024 Ladybird Srl - Via Leonardo da Vinci 16, 10126, Torino, Italy - VAT 10816460017 - All rights reserved